You are on page 1of 56

INTRO TO BB AND IMMUNOLOGY d.

Immune because of HBV vaccine


8. Valuable combination in therapy and follow-up
1. An iron-containing heme protein responsible for
in patients with germ cell tumor of the testes.
the peroxidase activity characteristic of
a. CEA and AFP
azurophilic granules: it accounts for the greenish
b. CA 125 and CA19-9
color of pus
c. AFP and B-hCG
a. Myeloperoxidase
d. HER-2 and B-hCG
b. Lactoferrin
9. Marker for gastric carcinoma:
c. Hemoglobin
a. CA 72-4
d. Myoglobin
b. CA 19-9
2. It binds iron which is an essential nutrient for
c. CA 125
microbes.
d. CA 15-3
a. Lactoferrin
10. A biomarker that exhibits greater specificity than
b. Hemoglobin
other proinflammatory markers in identifying
c. Myeloperoxidase
patients with sepsis and can be used in the
d. Myoglobin
diagnosis of bacterial infection
3. Large granular lymphocyte (LGL) is synonymous
a. Ceruloplasmin
with:
b. Cytokines
a. NK cell
c. Procalcitonin
b. B lymphocytes
d. CEA
c. T lymphocyte
11. What should be done if a noticeable clot is found
d. Cytotoxic T lymphocyte
in an RBC unit:
4. It is referred to as congenital thymic hypoplasia
a. Issue the unit; note the presence of a
which results from defective embryogenesis
clot in the release form
leading to a reduced (or absent) thymus.
b. Issue the unit: the blood will be filtered
a. Wiskott Aldrich Syndrome
c. Filter the unit in the blood bank before
b. Bruton's agammaglobulinemia
issue
c. Di George Syndrome
d. Do not issue the unit
d. X-linked agammaglobulinemia
12. The following dilutions were set up to titer an
5. Anti-dsDNA antibodies are associated with
antibody. The following results were obtained:
which of the following?
1:4 = +
a. CMV infection
1:8 = +
b. Syphilis
1:16 = +
c. SLE (Systemic Lupus Erythematosus)
1:32 = +
d. Hemolytic anemias
1:64 = (0)
6. It is a chronic progressive inflammatory
How should the titer be reported out?
autoimmune disease marked by progressive
a. 4
dryness of the eyes and the mouth
b. 16
a. SLE
c. 64
b. Rheumatoid arthritis
d. 32
c. Sjogren syndrome
13. The first line of defense in protecting the body
d. Scleroderma
from infection includes all of the following
7. Interpretation of HBV panel:
except:
HBsAg - Neg
a. Normal microbial flora
Anti-HBc - Pos
b. Unbroken skin
Anti-HBs – Pos (na dapat reactive)
c. Secretions such as mucus
a. Acute infection
d. Phagocytic leukocytes
b. Might be susceptible to a false positive
14. A primary function of the eosinophil is:
anti-HBc
a. Antigen recognition
c. Immune because of natural infection
b. Suppression of the inflammatory
response
c. Reacting in acute, systemic
hypersensitivity reactions
d. Phagocytosis
15. Class II MHC molecules are recognized by which
of the following:
a. CD4+ T cells
b. CD8+ T cells
c. B lymphocytes
d. NK cells

NOTES:

• Wiskott Aldrich Syndrome – part of hema,


microplatelets
• Bruton's agammaglobulinemia – cannot produce
immunoglobulins
• X-linked agammaglobulinemia – cannot produce
antibodies
• Anti-dsDNA – has lot of worms
• SLE – butterfly rash
• Scleroderma – thickening of the skin
• Vaccine only contains the surface antigen, so if
you get vaccinated against hepa B, magiging
immune ka sa anti-HBs
• Terminal cell tumors, trophoblastic tumors
(germ cell) – the behavior ng testicular cancer
introduces embryo equibolin since blastic cell
ang present ang narerelease ay pregnancy
markers (AFP and B-hCG)
• Carcino Embryonic Antigen (CEA) – used for the
monitoring of colorectral cancer
• CA 125 – used to monitor ovarian cancer
• CA 19-9 – used to monitor pancreatic cancer
• HER-2 – histopathologic marker of breast cancer
• Gastric carcinoma – present in duodenum or
sometimes pericardium of the stomach
• CA 15-3 – used to monitor breast cancer
• CA 72-4 – marker for gastric carcinoma
BB Q2 This is the kind of anemia characterized by a protein
band 4.1 deficiency.
This is the kind of anemia associated with a metastasis
of non myeloid cells to the bone marrow. o Both
o Hereditary elliptocytosis
 Myelophthisic anemia
o Neither
 Megaloblastic anemia o Hereditary spherocytosis
 Myelofibrosis
 Myelodysplastic syndrome Anemia of chronic renal disease is an anemia classified
by a deficiency in ______
What is the vitamin deficiency associated with
megaloblastic anemias o Vitamin B12
o Erythropoietin
o Thiamin o Sodium
o Niacin o Iron
o Pyridoxine
o Cobalamin Hemoglobin H disease results from:

This is an anemia characterized by a spectrin o Absence of 1 of 4 alpha genes


deficiency (not sure) o Absence of all alpha genes
o Absence of 2 of 4 alpha genes
 Hereditary spherocytosis o Absence of 3 of 4 alpha genes
 Hereditary elliptocytosis
 None of the above The lab finding on a patient are as follows:
 Hereditary pyropoikilocytosis MCV: 55 fL MCHC: 25% MCH 17pg
What test differentiates anemia of chronic disease vs. A stained blood film of this patient would most likely
iron deficiency anemia? reveal a red cell picture that is:
 AOTA o Macrocytic, hypochromic
 Transferrin saturation o Normocytic, normochromic
 TIBC o Microcytic, hypochromic
 FEPP o Microcytic, normochromic
Which of the ff. would most likely develop iron Excessive amount of iron that accumulate in the blood
deficiency anemia and tissues leading to injury in the parenchymal
tissues.
o 3 months pregnant mother
o Physically active young man o Hemosiderosis
o Colorectal carcinoma in situ patient o Sideroblastic anemia
o Patient who is a chronic smoker o Porphyria
o Hemochromatosis
What is the 1st therapy for patients with megaloblastic
anemia due to Diphyllobothrium latum infection (not This is a condition in which there is deficiencies of
sure) both CD55 and CD59 in the surface of the red cell.
o Treatment with praziquantel o PNH
o Replacement of folic acid o PCH
o PRBC transfusion o MDS
o Treatment with cyanocobalamin o CAD
What disorder is associated with erythrocytes that are
thermally instable and fragment when heated to 45 to
46 degrees celcius?

o Hereditary elliptocytosis
o Heridtary pyropoikilocytosis
o Hereditary spherocytosis
o PNH

Donath-Landsteiner antibody is present in patients


with _______.

o Paroxysmal Cold hematuria


o Diamond-Blackfan anemia
o Paroxysmal Nocturnal hemoglobinuria
o Pure red cell aplasia

This is a congenital anemia characterized by the


presence of webbing of digits. Café-au-lait spots in the
trunk and back, and congenital cardiac or renal
anomalies in patients. – Fanconi’s syndrome
BB LEC (1st week) o Blood-borne Disease Screening
o Compatibility Testing – cross match
INTRODUCTION TO BLOOD BANKING and BTRH (blood type & Rh test)
Blood Banking - Blood Transfusion
- Transfusion Reaction Investigation (If
- Collection, storage/preservation of blood and necessary) – pathologist, hematologist, RMT,
preparation of blood components patient
- Transfusion practices
o Compatibility testing RBC Biology and Preservation
o Component therapy - Normal chemical composition and structure
- AKA Immunohematology of the RBC membrane
History - Hemoglobin structure and function
- RBC metabolism
• 1492 – Pope Innocent VII; first ever recorded - RBC storage lesion
blood transfusion
• 1869 – Braxton Hicks; recommended sodium RBC Membrane
phosphate as a nontoxic anticoagulant
• 1901 – Karl Landsteiner in 1901 discovered the
ABO blood groups
• 1900's – Edward E. Lindemann; first to
successfully perform vein-to-vein transfusion
of blood
• 1900's – Unger; designed the syringe-valve
apparatus that made transfusions from
donor to patient by an unassisted physician
practical
• 1914 – Hustin; sodium citrate as an
anticoagulant solution for transfusions - RBC is composed of a double layer
• 1915 – Lewisohn; determined the minimum phospholipids w/ integrated protein and
non-toxic amount of citrate needed for carbohydrates (can be mixture or solo)
anticoagulation - Notable are:
o Protein band 4.1
• 1916 – Rous and Turner; introduced a citrate-
o Spectrin
dextrose solution for the preservation of
o Glycophorins – dito dumidikit ibang
blood
blood group
• 1930's (World War II) – Charles Drew;
▪ GP A – will carry the blood
developed techniques in blood transfusion
group M, N
and blood preservation and pioneer in the
▪ GP B – will carry the blood
establishment of a widespread system of
group Ss, U*
blood banks
- Spectrin, protein 4.1, ankyrin – RBC
• 1943 – Loutit and Mollison; formula for the
membrane proteins that are responsible for
preservative acid-citrate-dextrose (ACD).
the regulation of the shape of the cell
• 1947 – Publication of several papers on blood
- Protein band 4.1 deficiency – results in
preservation and establishment of blood
hereditary elliptocytosis
banks in many major US cities
- Spectrin deficiency – results in hereditary
• 1957 – Gibson; improved preservative
spherocytosis (hyperchromatic)
solution called citrate-phosphate-dextrose
- MCHC – average hemoglobin content of one
(CPD)
cell
Overview of Blood Bank Process o ↓ - hypochromic
▪ Micro, hypo – IDA, ACD
- Blood Donation (sometimes normo size),
- Blood Component Preparation and Storage thalassemia,
- Pre-transfusion Testing
hemoglobinopathies, lead ▪ responsible for oxygen
poisoning* carrying capacity of
o N – normochromic hemoglobin.
▪ Aplastic anemia ▪ Once it binds to hemoglobin,
o ↑ - hyperchromic there’s no choice for
▪ Hereditary spherocytosis – hemoglobin but to release
no central pallor the oxygen
- Deformability decreases with: ▪ More likely ↓ H+
o Decreased phosphorylation of concentration
spectrin due to loss of ATP o ↓ Temp, 02/P50
o Accumulation of membrane calcium o Hyperventilating patient - ↓CO2 =
(membrane rigidity and loss of nagiging basic yung pH
pliability) o pH – 7.35-7.45
▪ In patients with o pCO2 – 45-35
hypercalcemia that o HCO3 – 22-26
demonstrates anemia - When right shifted probable
possible of cell’s burst changes/possible of:
- Permeability o ↑ DPG, Temp, O2/P50, H+
o Permeable to water and anions concentration - ↓ pH
o Relatively impermeable to cations ▪ Inverse relationship of
▪ Na+, K+, ATPase pump – +
hydrogen (H ) & pH – the
maintain cations more H+ ions present, the
▪ ↑ intracellular K+ ; ↓ lower the pH
intracellular Na+ --- ↑ - Patient with hemoglobinopathies – shift to
extracellular Na +
; ↓ the left
+
extracellular K (Potassium in - Patient with metabolic or respiratory acidosis
Sodium Outside/PISO) – shift to the right

Hemoglobin RBC Metabolism

Hemoglobin dissociation curve – sigmoidal in - Anaerobic (glycolytic) pathway


appearance o generates about 90% of the ATP
needed by the RBC
- Approximately 10% is provided by the
accessory pathway:
o Pentose phosphate pathway or
Glutathione or Hexose
Monophosphate Shunt (MOST of the
10%) – responsible for preventing the
oxidation of the hemoglobin
▪ Glucose 6 Phosphate
Pathway – responsible for
glutathione synthase (gets
the free radicals inside our
cells) and kapag kinuha nya
yung free radicals walang
mag ooxidize sa hemoglobin
natin, therefore, reducing the
- When left shifted probable changes/possible
potential to oxidize the
of:
hemoglobin.
o ↑ Abn Hb
• Oxidation = Fe2+ inside
o ↑ pH
the hemoglobin and
o ↓ DPG (2, 3-diphosphoglyceric acid)
when it gets oxidize it
will turn into → Fe3+ Oxygen dissociation Shift to the left (increase
(Methemoglobin) curve in hemoglobin and
• Deficiency – G6PD oxygen affinity; less
deficiency, which results oxygen delivered to
in Heinz bodies (ferric tissues)
iron deposits in RBC) Plasma K+ Increased
deposition in the RBC Plasma hemoglobin Increased
because of unable to
reduce the potential to • shift to the left – increases oxygen affinity;
oxidize less oxygen delivered to tissues (gahaman
o Methemoglobin reductase pathway or yung hemoglobin)
Cytochrome b5 reductase pathway – • shift to the right – galante pero di nya kaya
responsible for reverting the present mag dala ng madami
oxidation • Plasma K+ - as the cell increases
▪ e.g., if there’s (naputol……)
methemoglobin present, • Newborns are given fresh RBC, too much or
what will cytochrome b5 do is too litle K+ is bad for the patient
to revert it from ferric to • Hemoglobin is very nephrotoxic – toxic in
ferrous kidneys
o Luebering-Rapoport shunt/pathway –
RBC PRESERVATION
responsible for the production of 2,3-
DPG (responsible for oxygen carrying Anticoagulant-Preservative Solutions
capacity)
NAME ABBREVIATION STORAGE TIME
(DAYS)

Acid citrate- ACD-A 21


dextrose
(formula A)*
Citrate- CPD 21
phosphate
dextrose
Citrate- CP2D 21
phosphate-
double-
dextrose
Citrate- CPDA-1 35
phosphate-
dextrose-
adenine
*ACD-A is used for apheresis components
RBC Storage Lesion
CPDA-1 – usually use in the lab
CHARACTERISTIC CHANGE OBSERVED
CHEMICAL FUNCTION PRESENT IN
% Viable cells Decreased
ACD-A CPD CP2D CPDA-1
Glucose Decreased Citrate Chelates X X X X
(sodium calcium;
ATP Decreased
citrate/citric prevents
Lactic acid Increased acid) clotting
Monobasic Maintains pH X X X X
pH Decreased sodium during
phosphate storage;
2,3-DPG Decreased necessary
for
maintenance pH (measured at 6.6 6.5 6.5
of adequate 37°C)
levels of 2,3- 24-hour 83 85.1 80
DPG survival*(%)
Dextrose Substrate for X X X X ATP (% initial) 68 67 68.5
ATP
production 2,3-DPG (% initial) 6 6 5
(cellular
Hemolysis (%) 0.5 0.7 0.6
energy)
Adenine Production X
of ATP
(extends Long Term Preservation (Freezing)
shelf-life
from 21 to 35 1ST – maselan (personal belief, rare blood type) konti
days) lang patient na may Rh null

AMA – against medical advice


Additive Solutions How to do freezing:
- Purpose: o Need to incorporate glycerol (anti-
o Extends shelf-life (from 35-42 days) freeze, freezes only RBC not plasma)
o Allows more platelets and plasma to o Glycerol – calcium oxalate
be harvested - Autologous donation and/or RARE blood
o Makes blood less viscous; easier to types
transfuse
- Glycerol:
▪ If blood is viscous, it’s harder o High-concentration: 40% w/v
to transfuse o Low-concentration: 20% w/v
- Licensed brands: (more likely used in private
hospitals – St. Luke’s, Makati Med., The ADVANTAGES DISADVANATAGES
Medical City)
Long-term A time-consuming process
o Adsol (AS-1; Baxter Healthcare)
storage (10
o Nutricel (AS-3; Pall Corporation)
years)
o Optisol (AS-5; Terumo Corporation)
Maintenance of Higher cost of equipment and
- Saline RBC viability and materials
- Adenine function
- Glucose Low residual Storage requirements (–65°C)
- Mannitol (AS-1 & 5) leukocytes and
- Citrate & phosphate (AS-3) platelets
Removal of Higher cost of product
NAME ABBREVIATION STORAGE
significant
TIME (DAYS)
amounts of
Adsol (Baxter AS-1 42 plasma proteins
Healthcare)
Nutricel (Pall AS-3 42 High VS Low Glycerol
Corporation)
ADVANTAGE HIGH GLYCEROL LOW
Optisol AS-5 42 GLYCEROL
(Terumo
Corporation) Initial freezing –80°C –196°C
temperature
Need to control No Yes
AS-1 AS-3 AS-5 freezing rate
Storage period 42 42 42 Type of freezer Mechanical Liquid
(days) nitrogen
Maximum –65°C –120°C
storage
temperature
Shipping Dry ice Liquid
requirements nitrogen
Effect of Can be thawed Critical
changes in and refrozen
storage
temperature

Current Trends

- Improved preservation methods and


pathogen elimination (e.g., alkylating agents
that interfere with pathogen nucleic acid)
- RBC Rejuvenation (Rejuvesol)
o Reviving/rejuvenating agent
▪ e.g., as long as the blood is
not clotted, put rejuvesol and
it can still be transfuse
o The only FDA-approved rejuvenation
solution for use with CPD, CPDA-1,
and CPD/AS-1
o Contains phosphate, inosine,
pyruvate and adenine
o RBCs stored in the liquid state can be
rejuvenated at outdate or up to 3
days after outdate
- RBC Substitutes
o AKA blood substitutes, oxygen
therapeutics or artificial oxygen
carriers (AOC), plasma expanders,
hemoglobin derivatives
(DI NYA DINISCUSS YANG 3 TABLES NA YAN, NASA
o e.g., If a patient has many history of
BOOK TO BTW)
transfusion reaction or has
autoantibodies (attack it’s own RBC,
result in hemolysis in vivo) in his/her
own RBCs
o Types:
▪ Hemoglobin-based oxygen
carriers (HBOCs) -
ultrapurified stroma-free
hemoglobin (SFH) of bovine,
human or recombinant origin
▪ Perfluorocarbons (PFCs) -
synthetic hydrocarbon
structures in which all the
hydrogen atoms have been
replaced with fluorine, and
are excellent gas solvents,
effective gas carriers
BB LAB (1ST WEEK) blood portion (plasma, rbc, wbc) will
then go back to the donor
OVERVIEW OF THE BLOOD BANK PROCESS o Deferral period for platelet apheresis
• Blood Donation is only 3 days then after 3 days he/she
• Blood Component Preparation and Storage can donate whole blood because it
• Pre-transfusion Testing only takes 3 days to recuperate the
• Blood Transfusion loss platelets.
▪ Resulting product – many
• Transfusion Reaction Investigation (If
platelets are harvested in one
necessary)
donor/unit
• Blood Bank Inventory, Maintenance and QA
▪ Best to transfuse to a patient
• Special Techniques
w/ platelet
Blood Donation thrombocytopenia is platelet
apheresis product because
Step 1: Educational Materials
there are more contents and
- Gauge 16,17,18 is used in blood donation (like is only exposed to one donor
straw of Zesto)
Blood Bag Centrifuge (Cryofuge) – for the
Step 2: The Donor Health History Questionnaire preparation of blood products from whole blood is
called cryofuge because it is refrigerated
- There are questions that a donor should
know what to answer, hindi ikaw ang mag
tatanong dapat si donor mismo ang sumagot

Step 3: The Abbreviated Physical Examination

- Should be done by a nurse or licensed


physician Blood Bag w/ Satellite Bags
Tests for Hepatitis:

- HBs Ag
o If the donor is 6-8 months into the
infection, walang madedetect na HBs
Ag pero infected sya, to detect it, it
he/she should be test to HBc Ag • 2 bag – FFP, PRBC
- HBe Ag • 3 bag – FFP, PC, PRBC
- HBc Ag • 4 bag – CPP, CP, PC, PRBC
- anti-HBs - Hindi tinatanggal ang mother bag, kadalasan
- anti-HBc PRBC and naiiwan sa mother bag
- anti-HBe - Every lipat ng bag dapat may label
Blood Component Preparation and Storage Pre-transfusion Testing
- Whole blood Blood-borne Disease Screening
o Packed RBC (frozen, washed,
leukocyte-reduced or irradiated) Current Donor Screening Tests for Infectious
Diseases
o Platelet or granulocyte concentrate
TEST DATE TEST REQUIRED
o Fresh plasma (cryoprecipitate AND
cryoprecipitate poor plasma) Syphilis 1950s
- Apheresis
o Upon extraction of blood into the Hepatitis B surface 1971
donor it will go to apheresis machine antigen (HBsAg)
then it will be separate the platelets Hepatitis B core 1986
into a single bag while the remaining antibody (anti-HBc)
Hepatitis C virus 1990 ▪ If there’s presence of
antibody (anti-HCV) hemoglobin in urine – acute
Human 19921 transfusion reaction
immunodeficiency ▪ If there’s presence of hives
virus antibodies (anti- and the result of CBC has
HIV-1/2) – HTLV III presence of hemophilia –
Human T-cell 19972 allergic transfusion reaction
lymphotropic virus
antibody (anti-HTLV- Blood Transfusion
I/II)
Human 1999 - Need 16-18 IV catheter filtered
immunodeficiency - Filter purpose
virus (HIV-1)(NAT)*,** o Is to filter blood clots in bag
Hepatitis C virus (HCV) 1999 o Clotted blood has activated platelets
(NAT) ** and when it enters the patient’s
West Nile virus (NAT) 2004 blood stream, it will result to
embolus, thrombus formation and
Trypanosoma cruzi 2007 will be deposited into pulmonary
antibody (anti-T. tract ending pulmonary embolism or
cruzi) brain stroke
• Malaria (common in Mindoro and Palawan)
• CMV (cytomegalo virus) Transfusion Reaction Investigation

Compatibility Testing - Collection of clinical samples


o Patient post transfusion urine
• ABO and Rh Typing o Patient pre transfusion blood
• Cross-matching o Patient post transfusion blood
o Major (I) – patient serum vs. donor o Donor blood
RBC - Direct and indirect Coomb's test - determine
▪ Done to test if it’s safe to the nature of antibody
transfuse the RBC of the o Direct lies inside
donor to patient’s serum and o Indirect – chronic (???)
to avoid the acute hemolytic - Repeat compatibility testing
transfusion reaction o Using pre transfusion blood and
▪ ABO incompatibility – donor blood
deadliest incompatibility
o Minor (i) Blood Bank Inventory, Maintenance and QA
• Minor Blood Group Typing - Blood donor registry
*** Tests done in the PH (red colored fonts) - Blood component inventory
- Reagent preparation and QA
Blood Transfusion o Everyday monitoring
- Instrument monitoring and QA
- Clerical checks
o Everyday monitoring of ref temp.,
- Blood transfusion instruments
- Rate of administration centrifuge, freezer, reagents, blood
component inventory, etc.
o PRBC (packed RBC) should be
transfused within 2 hrs Special Techniques
- Clinical indicators of adverse transfusion
reactions - HLA typing
o Done by nurses o Organ donation
o If there’s symptom of pain automatic o Tumor markers
stop the transfusion then report to o Types of hypersensitivities
the lab (clerical checks, lab works – - Paternity testing
patient’s urine, pre & post o Should be under Human Leukocyte
transfusion blood, blood from donor) Antibody (HLA) testing
BB LEC (2nd week) syndrome, patients taking
antacids.
ANEMIAS Irons is well absorbed in an
acidic environment. You need
- Anemia is a decreased in the oxygen carrying
an acidic environment for
capacity of the blood
iron to be absorbed readily.
o It can arise if there is insufficient
Antacids – lowering or
hemoglobin or the hemoglobin has
making the acidic
impaired function.
environment to become
IRON AND HEME DISORDERS basic. Therefore, iron is not
absorbed well
Iron Deficiency Anemia o Chronic blood loss
▪ Chronic hemorrhage/bleeding can
- One of the most common anemias present in a
also result to IDA
whole clinical practice
▪ Ex. Tumors, hemorrhoids, bladder
- This is the type of anemia that arises due to a
stones, kidney stones, PNH.
patient being deficient in iron.
NOTES: constant loss of RBC,
- It can occur due to many reasons:
hemoglobin = constant need
o Inadequate intake
to replace it
▪ Iron deficiency anemia can develop
when the erythron is slowly
starved of iron
▪ Seen in patients who have a poor
diet
▪ It will result in a slow developing
iron deficiency
NOTES: e.g., patients who do
not eat iron rich foods like
vegans who do not eat red
meats
o Increased need
▪ Iron deficiency can develop when
the level of iron intake is
inadequate to meet the needs of Stage 1: Iron depletion
an expanding erythron - Storage iron ay paubos lang
▪ Can be seen in rapidly growing
children Stage 2: Iron deficient erythropoiesis
NOTES: e.g., lactating
- Unti unti nang nababawasan yung iron sa
mother, why they increase
erythropoiesis or erythron iron
need? Because they need to
rump up RBC production Stage 3: Iron deficiency anemia (most chronic stage)
therefore exhausting the iron
- All of the storage and RBC iron are deficient or
stores
few
o Impaired absorption
▪ Even when diet is adequate in iron, LABORATORY DX/DIAGNOSIS
the inability to absorb that iron
through the enterocyte into the - Screening Tests for IDA
blood will result in IDA o CBC: anisocytosis, microcytosis,
▪ Medications: antacids hypochromia
(omeprazole, proton pump ▪ Declining MCV. MCH. MCHC levels
inhibitors, h2 inhibitors), etc. (first need to see for any kind of
NOTES: happens with anemia work up are these the RBC
patients with malabsorption indices, because this will give you a
lot of clues)
▪ Poikilocytosis codocytes and Some patients demonstrate
elliptocytes thrombocytosis because of high EPO
NOTE: IDA is a microhypo
kind of anemia. Small cell
with diminished hemoglobin
concentration. Anisocytosis
because of variable sizes.
- Diagnosis of IDA - PBS slide of a patient with iron deficiency that
o Serum iron: decreased – because it is a recently have a transfusion. Also known as
direct indicators of iron levels dimorphic kind of anemia because there’s
o TIBC: increased microhypo, hypo, normo, spherocytes
o Serum ferritin: decreased – because it is a - Dimorphic anemia happens due to transfusion
direct indicators of iron levels and sideroblastic anemia
o Free erythrocyte protoporphyrin: TREATMENT
increased because protoporphyrin IX
requires iron to become heme. If the - First therapy is to treat what is causing the loss
patient doesn’t have iron, then of iron, or inadequacy of iron
protoporphyrin IX will probably increase - If nutritional deficiency, oral supplements of
and accumulate in the RBC. Is it ferrous sulfate and if allergic to it then ferrous
functional? NO. gluconate, if allergic to either or then ferrous
fumarate for not more than 6 months should be
NOTES: adequate because it will cause blackening of
Low transferrin = high TIBC because the stool therefore a false diagnosis of
what is measured in TIBC is what iron gastrointestinal bleeding
it can carry. If the transferrin is - If severe IDA needs to be treated, blood
hungry for iron, then it will eat many transfusion is an option
in the testing, therefore the result o In chronic transfusions, an iron chelating
indicates that the patient has medication needs to be concurrently
insufficient iron administered (deferoxamine,
desferrioxamine – these drugs chelates
Megakaryopoeisis – formation of your iron in order to prevent
platelets hemochromatosis) to avoid iron overload
- NOTES: first line – replace what is missing
Erythropoietin – hormone RESPONSE TO RX/TREATMENT
responsible for the production of
RBC, produce by the kidney and liver. - Reticulocyte hemoglobin content will correct
EPO is increased in iron deficiency but w/n 2 days but the microhypo will not correct in
in the first stages why 2 days but rather months
thrombocytosis happens? Because - Reticulocyte count begin to increase within 5 to
EPO doesn’t play any role 10 days (there’s already symptomatic
improvement of the patient)
In the first stages of the - PBS will show a biphasic population
megakaryoblast (MB) formation – o Normocytic, normochromic
the first hormone to play is the EPO. o Microcytic, hypochromic
In order for the megakaryoblast to
Anemia of Chronic Disease/Inflammation (ACD)
mature, it needs EPO tapos dito na
papasok si TPO - This kind of anemia is associated with systemic
diseases (because there are acute phase
Platelet is high in patients with IDA is reactants possible for iron metabolism)
due to EPO playing a major role in the including inflammatory conditions (i.e. RA,
megakaryoblast maturation. SLE), chronic infections (i.e. TB, HIV/AIDS), and
malignancies.
Pathogenesis:
Increase synthesis of IL-3 = increase platelet
- Anemia of chronic inflammation is due to the
formation
body storing iron in response to infection.
(since bacteria need iron, the bacteria will store TREATMENT
the iron but since there’s no agent, your body
- Therapeutic administration of EPO can correct
becomes the collateral damage)
ACD, but iron must be administered
- It is controlled by 3 acute phase reactants:
concurrently (not usually use because EPO is an
o Hepcidin
intramuscular injection, iniinject sa abdomen ng
▪ This is a hormone that is produced
patient)
by the hepatocytes
- Anemia is typically not severe
▪ It regulates body iron levels,
particularly absorption of iron in NOTES: since in inflammation also exerts a decrease
the intestines and release from in EPO synthesis, you also need to give EPO
macrophages
For treating ACD, you need to first treat the causative
o Lactoferrin agent.
▪ This is an iron-binding protein in the e.g., patient has rheumatoid arthritis, you should give
granules of neutrophils him/her methotrexate and anti-inflammatory drugs
▪ During inflammation, lactoferrin is in order to suppress the inflammation because if you
released into the plasma, leading to do not suppress the causative agent, it’s a never-
low iron levels. ending cycle
o Ferritin
▪ This is the main storage of iron, in Sideroblastic Anemia
inflammation ferritin is increased - These are diseases that interfere with
leading to more iron stored. production of adequate amounts of
LABORATORY DX/DIAGNOSIS protoporphyrin
- Anemia may be microcytic and hypochromic
- Mild anemia, with hemoglobin concentrations - In contrast to IDA, Fe is abundant in the bone
usually 8 to 10 g/dL (tolerable) and w/o marrow
reticulocytosis o It is present in the mitochondria,
- Iron studies show low sFe and TIBC denoting a deficiency in protoporphyrins
- Ferritin, Hepcidin are markedly increased - Erythroblasts are positive in Perl's Prussian Blue
- ESR is markedly increased stain (iron stain)
- Platelets may be normal or markedly NOTE: kapag maraming blue, maraming iron
increased***
What can cause SA?
NOTES: why TIBC is low in ACD? Kase naka - Hereditary
tago na sa kanya, ferritin and transferrin are - Acquired
part of iron metabolism. If di sya gutom, naka o RARS (Refractory Anemia with Ringed
tago lang sa kanya. Bakit naka tago sa kanya? Sideroblasts) – kind of myelodysplastic
Because there’s an inflammation. syndrome
o Due to drugs:
The lower the RBC = the higher is the ▪ Antitubercular drugs (Isoniazid)
inflammation ▪ Chloramphenicol (important, most
common antibiotic that is
Platelets is increased in ACD because IL-3 is a associated with aplastic and
multiclass growth factor, interleukin increase sideroblastic anemia. For patients
their concentration in inflammatory taking this, it is myelotoxic – toxic
episodes. Let say patient is chronically in bone marrow)
inflamed, hindi tumataas and RBC kasi nga ▪ Alcohol
kulang ang iron pero tumataas ang platelets ▪ Lead
kasi inflamed.
▪ Chemotherapeutic Agents (there is no parenchymal/significant
(Doxorubicin, Vinblastine, etc.) damage)
o Hereditary (Hemochromatosis – there is
LEAD POISONING
tissue damage)
- This is of special concern in children because of
HEMOSIDEROSIS
the CNS effects, as well as hematologic effects
- Lead interferes with porphyrin synthesis at - Minimal to no tissue damage
several steps, but the most critical are: - Iron only accumulates on the mature erythron
o Conversion of ALA to porphobilinogen
HEMOCHROMATOSIS
o Incorporation of Fe to protoporphyrin IX
- Anemia, when present, is most often - With moderate to severe tissue damage
normocytic and normochromic (acute in - Iron accumulates on the erythroblasts up to the
nature) mature erythron and organs (bronze skin)
- A classic finding is basophilic stippling (lead
poisoning) Pathogenesis:
- Treatment is administration of EDTA to chelate - Increased Iron in the serum leads to multiple
Pb. organ damage
o Cardiovascular
NOTES: lead poisoning is a kind of SA or o Hepatic – because iron is stored in the
porphyria, special concern because of CNS liver
effect. o Pancreatic – it interferes with the ducts
function, acute pancreatitis
San nakukuha? Sa lead paints, lead o Nephrotic – kidneys are the ones that
contaminated water, surfaces. Because filters
these are readily absorbed by skin. CNS effect
of this is mental retardation. TREATMENT

- Treatment for secondary tissue damage, such


Lead are found sa mga toys sa Divisoria, as liver cirrhosis and heart failure, follows
because their paints are cheap kase mura standard protocols
lang din ang lead paint - For hemochromatosis, therapeutic phlebotomy
PORPHYRIAS provides a simple, inexpensive, effective means
of removing Fe
- Porphyrias are inherited disorders of - For hemosiderosis, deferoxamine (iron
deficiencies in enzymes responsible for the chelating) is the drug of choice.
production of heme
- Due to a deficiency of an enzyme, the substrate MEGALOBLASTIC ANEMIAS
accumulates, resulting to toxicity. (These are
ETIOLOGY:
the patients that should not expose to UV lights
because of these..) - The root cause of megaloblastic anemia is
o Hematologic impaired DNA synthesis
o Dermatologic - The anemia is named for the very large cells of
o Neurologic the bone marrow due to reduction of cell
divisions
Iron Overload

- Excess accumulation of iron results from NOTES: There’s an asynchronous maturation.


acquired or hereditary conditions in which the The cell size in megaloblastic hindi makapag
rate of acquisition exceeds the rate of decrease kasi yung nuclear stage nya
utilization and release immature pa dahil sa DNA synthesis.
- It can be acquired or hereditary
MAJOR FACTORS:
o Acquired: Chronic transfusion (IDA,
Thalassemia) leading to Hemosiderosis Vitamin B12

- Also known as cobalamin


- Physiologic role: ▪ Loss of epithelium along the GI
o Vitamin B12 is a coenzyme in two tract can result in gastritis, nausea,
biochemical reactions in humans or constipation
▪ Isomerization of methylmalonyl o CVD
coenzyme COA
▪ Transfer of a methyl group from 5- NOTES: manifestation in alimentary canal is
meth (5-methyl THF) to atrophic glossitis (absence of the filiform
homocysteine, which methionine papillae, nawawala yung papillae ng dila)

VITAMIN B9 CAUSES OF VITAMIN DEFICIENCY (FOLATE)

- Also known as folate or folic acid - INADEQUATE INTAKE


- Physiology: o Folate is numerous in green leafy
vegetables
- INCREASED NEED
o Increased need of folate occurs during
pregnancy and lactation; Infants and
children also need increased folate for
growth
- IMPAIRED ABSORPTION
o Sprue, celiac disease, deficiency in the
folate transport protein
- IMPAIRED USE OF FOLATE
o Numerous drugs impair folate
metabolism (i.e. Methotrexate, 5-FU,
Vitamin B12 and B9 – both play a role in conversion of Hydroxyurea, etc.)
the uridylyl (dUMP) to thymine (dTMP) - EXCESSIVE LOSS OF FOLATE
o Physiologic loss occurs in the kidneys
NOTES: since uridylyl is present in the RNA and (dialysis who have chronic kidney
thymine is present in the DNA, if you cannot convert disease)
uridylyl to thymine hindi ito mag poproceed sa DNA
maturation CAUSES OF VITAMIN DEFICIENCY (VITAMIN B12)

OTHER CAUSES OF MEGALOBLASTOSIS OR - INADEQUATE INTAKE


MACROCYTOSIS o Best dietary sources of B12 are animal
products
- Myelodysplastic Syndromes - INCREASED NEED
- Congenital Dyserythropoietic anemia o Increased need occurs during pregnancy,
- Acute Erythroid Leukemia (AML M6) lactation, and growth
- Antiretroviral drugs – Reverse transcriptase - IMPAIRED ABSORPTION
inhibitor o Lack of IF (Pernicious anemia), lack of
o Responsible for converting RNA to DNA haptocorrin, etc.
SYSTEMIC MANIFESTATIONS OF DEFICIENCY - FAILURE TO SEPARATE B12 FROM PROTEINS
o Failure to separate B12 from proteins
- When DNA synthesis and subsequent cell occur during increased pH or decreased
division are impaired by lack of folate or B12 acid secretion, hypochlorhydria (cannot
- Systemic symptoms include: produce acid and cannot do separation,
o Symptoms related to anemia (there’s produces slightly alkaline gastric juice is
palor) produced by the patient)
▪ Fatigue, weakness, and shortness - FAILURE TO SEPARATE B12 FROM
of breath HAPTOCORRIN
o Symptoms related to alimentary canal o Achlorhydria or lack of trypsin due to
▪ Loss of epithelium on the tongue chronic pancreatitis
results in smooth surface and - LACK OF INTRINSIC FACTOR (IF)
soreness
o Pernicious anemia (autoimmune o Howell Jolly Bodies
diseasese targeting cells that produce o Hypersegmented Neutrophils
intrinsic factors like parietal cells), Loss of - Occasional NRBCs, basophilic stipples, and
parietal cells with H. pylori infection, total cabot rings may also be observed.
or partial gastrectomy, or hereditary IF
WBC Manual Differential
deficiency.
NOTE: B12 cannot be absorb without - Hypersegmentation of neutrophils is essentially
intrinsic factors pathognomonic for megaloblastic anemia
- OTHER CAUSES: - Not all cases of megaloblastic anemia will have
o Malabsorption hypersegmented neutrophils
o Imerslund-Grasbeck syndrome (decrease
endocytosis of IF-Cobalamin complex - NOTE: hypersegmented neutrophils is
rare) suggestive of megaloblastic
o D. latum infection
o Blind loops (stenotic intestines or loop Bilirubin and LDH
syndrome as a result of surgery or - Because many RBC precursors die during
inflammation) division in the bone marrow many RBCs never
enter the circulation.
NOTES: blind loops – hindi pa alam - The usual signs of hemolysis are evident,
kung nasaan including an elevation in the levels of Total Bili,
Pernicious Anemia B1, and LDH (LD6)

- Pernicious anemia is due to an autoantibody NOTES: since there is an increase turnover of


against the chief cells of the stomach or cells because of bursting due to incorrect
autoantibodies against IF. The chief cells maturation, ma-sesense ito ng katawan natin
produce HCI and IF in response to gastrin kaya sisirain nya ito
LAB DIAGNOSIS Bone Marrow Exam
- Screening test - The erythroid precursors show nuclear-
o CBC cytoplasmic asynchrony
o Reticulocyte Count o The nucleus doesn't match the
o WBC manual differential cytoplasmic picture of the cell (e.g.,
o Serum bilirubin nucleus is basophilic normoblast but RBC
o Serum LDH cytoplasm is polychromatic, doesn’t
- Specific diagnostic test match)
o Bone Marrow Examination - The WBCs also appear larger than normal
o Vitamin Assay - Megakaryocytes do not show consistent
o Gastric Fluid Analysis changes in megaloblastic anemia
o Antibody Assays
o Holotranscobalamin Assay Vitamin Assay
o Deoxyuridine Suppression Test - Assays for Vitamin B9, B12, Methylmalonic acid,
o Stool Analysis for Parasites and homocysteine should be included in the
CBC and Reticulocyte Count workup for megaloblastic anemia

- Patients with uncomplicated megaloblastic NOTE: tingnan kung ano yung kulang then
anemia are expected to have decreased Hgb yun ang palitan
and Hct, pancytopenia, and reticulocytopenia
- MCV is usually 100 to 150 fL and commonly >120
fL
- RDW is increased
- OM, HJ, HN: (classic findings)
o Oval macrocytes
ALGORITHM FOR VITAMIN ASSAY - MCV usually ranges from 100 to 110 fL
- Lacks hypersegmentation
- Seen in:
o Liver disease
o Chronic alcoholism
o Bone marrow failure
- Macrocytosis due to reticulocytosis

TREATMENT

- Treatment should be directed at the specific


vitamin deficiency and should include
addressing the cause of deficiency
Gastric Analysis and Serum Gastrin - Vitamin B12 IM in patients with pernicious
anemia to bypass the IF deficiency
- Gastric analysis may be used to confirm
achlorhydria BONE MARROW FAILURE SYNDROMES
- As a result of gastric achlorhydria, serum gastrin
levels can be markedly elevated Aplastic Anemia

- Aplastic anemia is a pancytopenia with bone


NOTES: work up for pernicious because if you
marrow hypocellularity
have autoantibodies against your chief cells,
- Aplastic anemia can either be:
wala kang napoproduce na HCL resulting
o Acquired/Idiopathic/latrogenic: Drugs
achlorhydria and an increase serum gastrin
(chloramphenicol), chemotherapy,
Holotranscobalamin Assay radiation
o Congenital: Fanconi, Diamond Blackfan,
- Holotranscobalamin is the metabolically active
etc.
form of vitamin B12
- Vitamin B12 assay is a more specific than
NOTES: aplastic – there’s no tissue
holotranscobalamin, therefore it is not adopted
Idiopathic – doesn’t know the cause
to a routine clinical algorithm.
Iatrogenic – cause by a medical
Deoxyuridine Suppression Test procedure

- Principle: ETIOLOGY:
o Preincubation of normal BM with
- Radiation
deoxyuridine will suppress the
o Marrow aplasia is a major acute sequela
subsequent incorporation of labeled
of radiation. Radiation damages DNA.
thymidine into DNA because the normal
(Who? Patients with cancer)
cells can successfully methylate uridine
- Chemicals
→ thymidine
o Benzene (clearing agent) is a notorious
o In patients with either a vitamin B12 or B9
cause of bone marrow failure.
deficiency, this suppression is abnormally
- Drugs
low.
o Chloramphenicol
Stool Analysis - Infection
o Hepatitis is the most common preceding
- A stool analysis for the presence of eggs or infection
proglottids of Diphyllobothrium latum can be o Aplastic anemia can rarely follow IM
supportive of the diagnosis of megaloblastic (infectious mononucleosis – caused by
anemia. CMV)
Macrocytic Nonmegaloblastic o Parvovirus causes transient aplastic crisis
in hemolytic anemias in infants and other
- Macrocytic nonmegaloblastic anemias are Pure Red Cell Aplasias
macrocytic anemias in which DNA synthesis is - Immunologic diseases
unimpaired
o Aplasia is a major consequence of TA- Shwachman-Diamond syndrome
GVHD (Transfusion Associated Graft
- Presentation is early in life with neutropenia
Versus Host Disease)
with pancreatic insufficiency and
o Eosinophilic fasciitis
malabsorption that leads to marrow hypoplasia
o Thymoma and
- Most patients have compound heterozygous
hypoimmunoglobulinemia.
mutations in SBDS that may affect both
o SLE (Systemic Lupus Erythematosus)
ribosomal biogenesis and marrow stroma
function.
NOTE: anti-dsDNA is usually present
in SLE PATHOPHYSIOLOGY
- Pregnancy
o Aplastic anemia very rarely may occur and - Bone marrow failure results from severe
recur during pregnancy and resolve with damage to the hematopoietic cell
delivery or with spontaneous or induced compartment
abortion - In aplastic anemia, replacement of the bone
- Paroxysmal Nocturnal Hemoglobinuria marrow by fat is apparent in the morphology of
o An acquired mutation in the PIG-A gene the biopsy specimen and magnetic resonance
resulting in a deficiency of CD55 (Decay imaging (MRI) of the spine
accelerating factor) and CD59 - Cells bearing the CD34 antigen are greatly
(Membrane Inhibitor of Reactive Lysis) in diminished. and in functional studies,
RBCs, causing attachment of C3b to the committed and primitive progenitor cells are
RBC resulting to activation of the virtually absent
complement pathway, leading lysis. DRUG INJURY
▪ NOTES: Tuwing gabi kapag
natutulog ka, nagiging acidic yung - Extrinsic damage the marrow follows massive
dugo mo, ang complements nag physical or chemical insults such as high doses
aactivate sa acidic environment, of radiation and toxic chemicals
naglilyse yung RBC, umiihi ka ng IMMUNE-MEDIATED
hemoglobin
o Treatment: Eculizumab (monoclonal anti- - Increased numbers of activated CD8+ T cells are
C5) observed in aplastic anemia patients and
usually decline with successful
CONSTITUTIONAL DISORDERS immunosuppressive therapy
Fanconi anemia LABORATORY DIAGNOSIS
- Autosomal recessive disorder, manifests as - Blood
congenital developmental anomalies, o Smear shows large RBCs and a paucity
progressive pancytopenia, and an increased risk (deficiency) of platelets and granulocytes
of malignancy. o MCV is commonly increased, <120 fL
- Patients with Fanconi anemia typically have o Reticulocytes are absent or few
short stature, café au lait spots (may mga pachi- o Presence of immature myeloid
pache yung patient), and anomalies involving precursors may suggest AML or MDS
the thumb, radius and genitourinary tract. - Bone Marrow
- The most common type. Type A Fanconi o “Dry tap” – fibrous
anemia, is due to a mutation in the FANCA gene. o Biopsy is fatty and has low cellularity
Dyskeratosis congenita TREATMENT
- Characterized by the triad of mucous - Hematopoietic Stem Cell Transplant (patient
membrane leukoplasia, dystrophic nails, <50 years old)
reticular hyperpigmentation, and with the - Immunosuppression (patient >50 years old)
development of aplastic anemia in childhood. - Blood Transfusion (if >50, deferoxamine must
- Due to mutations in the genes of the telomere be administered to avoid hemochromatosis)
repair complex
Pure Red Cell Aplasia

- This is a kind of marrow failure syndrome that is


more restricted to a single circulating cell type,
this type is the RBC
- This is characterized by anemia,
reticulocytopenia, and absent or rare erythroid
precursor cells in the bone marrow
- In adults, PRCA is acquired. In children, PRCA is
congenital (Diamond-Blackfan anemia)

Persistent Parvovirus Infection LABORATORY STUDIES

- Chronic parvovirus infection is an important, - Blood


treatable cause of PRCA o Anemia is present in most cases, either
- Parvovirus causes a transient aplastic crisis and alone or as part of bi or pancytopenia
an abrupt but temporary worsening of the o Macrocytosis is common
anemia due to failed erythropoiesis o Neutrophils are hypogranulated
- Bone marrow shoes red cell aplasia and the - Bone Marrow
presence of giant pronormoblasts. o Bone marrow is usually normal or
hypercellular
TREATMENT: PRCA o No single characteristic feature of
- Red cell aplasia is compatible with long-term marrow morphology distinguishes MDS
survival with supportive care alone
Myelophthisic Anemia
o RBC transfusions and iron chelation
- For persistent parvovirus B19 infection, almost - Myelophthisis, or secondary myelofibrosis, is
all patients respond to IVIg (intravenous reactive.
immunoglobulin) o Fibrosis can be a response to invading
tumor cells (metastasis)
Myelodysplasia
- Anemia is dominant in secondary myelofibrosis,
- The myelodysplastic syndromes are usually normocytic and normochromic.
heterogeneous group of hematologic disorders - WBCs are often elevated, sometimes mimicking
broadly characterized by both: a leukemoid reaction, with myelocytes and
o Cytopenias (1) due to bone marrow myeloblasts
o A high risk of development of AML (2)
- Five entities were defined:
HEMOLYTIC ANEMIAS: INTRINSIC AND
o Refractory anemia (RA) EXTRINSIC CAUSES
o Refractory anemia with ringed
Basics Of Hemolytic Anemias
sideroblasts (RARS)
o Refractory anemia with excess blasts - It can be due to either:
(RAEB) o Extrinsic defects
o Refractory anemia with excess blasts in o Intrinsic defects
transformation (RAEB-t) - Peripheral blood may contain the following:
o Chronic myelomonocytic leukemia o Schistocytes
(CMML) o Spherocytes
o Inclusions
NOTE: myelodysplasia – bone marrow - S/Sx are usually appropriate for hemolytic
desynchrony tissue episodes, jaundice, splenomegaly, gallstones

Dysplasia – asynchrony of the tissue NOTES: What protein decreases in patients


with hemolytic anemias? – haptoglobin
MDS ending is Acute Myeloid Leukemia (sequesters free heme in plasma)
(AML) – pre-leukemic manifestation
Heme is nephrotoxic
INTRINSIC DEFECTS - Mechanical – also known as march
hemoglobinuria
- Hereditary Spherocytosis
o Due to a deficiency of either spectrin (a or Autoimmune Hemolyric Anemia (AIHA)
B), ankyrin, or band 3
- This happens once a red cell is coated by an
o Predominant cells are spherocytes
autoantibody
o Diagnostic strategies:
o IgM - intravascular hemolysis
▪ Family history
(complement activation)
▪ MCHC >36 pg
o IgG-extravascular hemolysis (spleen
▪ Peripheral Blood Smear
culling)
▪ Osmotic Fragility Test
- Diagnostic test for AIHA is Coombs’s test
▪ SDS-gel electrophoresis for RBC
membrane proteins Paroxysmal Cold Hemoglobinuria (PCH)
o Treatment: No causal treatment for HS
- Hereditary Elliptocytosis - Presence of autoanti-P so called Donath-
o Due to a mutation in either the Spectrin Landsteiner antibody
(a or B), or Band 4.1 gene - Usually triggered by a viral infection
o Some mild or even asymptomatic cases - Diagnostic: (+) for Donath-Landsteiner
have nearly 100% elliptocytes, whereas in antibody
several cases, all kinds of bizarre Cold Agglutinin Disease (CAD)
poikilocytes can predominate
- Pyruvate Kinase deficiency - Chronic AIHA that usually affects the elderly
o The most common enzyme deficiency in - The antibody is usually IgM; usually it has an
the Embden-Meyerhof Pathway. anti-I specificity
o Newborns with PK def. present with - Presence of agglutination in PBS may indicate
neonatal jaundice; CAD
▪ Jaundice persists, accompanied - Blood transfusion is not very effective
with a very high reticulocytosis - Immunosuppressive drugs can reduce the titer,
o Anemia is variable in severity. but clinical efficacy is limited
- G6PD Deficiency - Management of CAD has changed due to the
o The most common enzyme deficiency in advent of rituximab discovery (anti-CD20)
the Hexose Monophosphate Shunt. Paroxysmal Nocturnal Hemoglobinuria (PNH)
o Manifests with abundant schistocytes,
bite cells (pre-splenectomy), Heinz - This is an acquired chronic HA characterized by
bodies (post-splenectomy) persistent intravascular hemolysis subject to
recurrent exacerbations
NOTE: What is the most common enzyme - RBCs lack CD55 (DAF) and CD59
deficiency in the world? – G6PD deficiency - TRIAD: Hemolysis, Pancytopenia, Distinct
tendency to venous thrombosis.
What is the most common non-G6PD? – - Screening Test: Sucrose Hemolysis test, Sugar
Pyruvate Kinase water test, Ham's acidified serum test
- Diagnostic Test: Flow Cytometry (CD55, CD59)
EXTRINSIC DEFECTS - Treatment: Eculizumab (anti-C5)
- Immunologic Malaria
o AIHA
o PCH - Caused by Plasmodium spp., most dangerous
o CAD falciparum
o PNH - Characterized by paroxysms: Chills, fever,
- Infectious sweating
o Malaria - Gold standard diagnostic test:
o Babesia o Thick and thin blood film
o HUS - Immunoassay test:
o Detection of parasitic LDH (Plasmodium Sickle Cell Crises
spp.) and HRP-II (P. falciparum)
- Crises are defined as the patients experience
- Treatment: Primaquine, Chloroquine
acute onset pain, fever, tachycardia, anxiety
- All organ systems can be affected by crises:
NOTES: Found in Palawan and Mindoro, part
(hypoxic and schemic)
of donor deferral
o Cardiothoracic: Acute chest syndrome
Babesiosis o Nervous: Chronic subacute central
nervous system damage.
- Caused by Babesia microti
o Skeletal: Aseptic necrosis
- Less severe than malaria
o Male Reproductive: Priapism
- Transmitted by Ixodes ticks
- Major cause of death: Opportunistic infection
- Diagnostic test: thick and thin blood film
due to vaso-occlusion
- Treatment: Azithromycin/Atovaquone
SICKLE CELL DIAGNOSIS
Hemolytic Uremic Syndrome (HUS)
- CBC
- Caused by either E. coli O157:H7 (grows in
- PBS
SMAC), or Shigella spp. (nonmotile)
- Hemoglobin electrophoresis / mass
- Characterized by hemolysis and renal failure
spectroscopy
(azotemia)
- Usually occurs following bloody diarrhea TREATMENT

HEMOGLOBINOPATHIES AND - Analgesia


- Hydroxyurea
THALASSEMIA SYNDORMES
o Mainstay therapy for patients with severe
Hemoglobinopathy symptoms

- Basic structure of hemoglobins: Hb C Disease


o Hb A - a2B2
- Caused by a mutation in the ß-globin gene that
o Hb A2-a2D2
changes the sixth amino acid from glutamic acid
o Hb F-a2y2
to lysine (a2B2 6Glu-Lys → glutamic acid to
- Major hemoglobinopathies:
lysine)
o Sickle Cell Syndromes
- Causes inclusions characterized as hexagonal or
o Hb C disease
"Washington Monument" crystals
o Methemoglobinemia
- Hb C may interact with Hb S to form Hb SC
Sickle Cell Syndromes crystals.

- Caused by a mutation in the B-globin gene that Methemoglobinemia


changes the sixth amino acid from glutamic acid
- Methemoglobin is generated by oxidation of
to valine (a2B2 6Glu-Val)
the heme iron moieties to the ferric state
- HbS polymerizes when deoxygenated to form a
- The blood has a characteristic bluish-brown
gelatinous network of fibrous polymers
muddy color
- Sickle Cell Trait - HbAS
- Congenital methemoglobinemia arises from
- Sickle Cell Anemia – HbSS
globin mutations that stabilize iron in the ferric
Sickle Cell Formation state or from mutations that impair enzymes
that reduce MetHb to Hb
- RBCs sickle when exposed to hypoxic - Acquired methemoglobinemia is caused by
environments, forming drepanocytes or sickle toxins
cells. - Treatment: IV/PO methylene blue (1 mg/kg) or
- When RBCs sickle, Ca+2 rushes in to the cell and ascorbic acid
K+ leaks out, causing pseudohyperkalemia
- Sickle cells cause plugs that cause Thalassemias
microinfarctions of organ systems, leading to
- The thalassemia syndromes are inherited
multiple organ failure.
disorders of a- and B-globin biosynthesis
- The reduced supply of globin diminishes o GOLD STANDARD FOR DIAGNOSIS:
production of hemoglobin tetramers, causing HEMOGLOBIN ELECTROPHORESIS
hypochromia and microcytosis. (AGAROSE GEL PH 8.6)
- Quantitative hemoglobin disorder
PREVENTION
B-Thalassemia Syndromes
- Genetic counseling for a and 13 thalassemia
- Heterozygotes, ß-thalassemia minor, anemia is carriers, homozygotes, and heterozygotes
minimal. - Antenatal diagnosis via PCR of fetal DNA to
- Homozygotes, B-thalassemia major, causes detect mutations in the globin genes.
accumulation of highly insoluble unpaired a
TREATMENT
chains
o Forming toxic inclusions in erythroblasts - Supportive therapy
o The shortening of RBC life span produces
a response by the kidney to release EPO
o Increased EPO, Increased erythropoiesis,
Extramedullary erythropoiesis
(hepatosplenomegaly)

a-Thalassemia Syndromes

- a Thalassemia Trait
o Asymptomatic, silent carrier state (-a/aa)
- HbH Disease (Heterozygous a-thal 1 and 2)
o Causes accumulation of unpaired y chains
(Hb Barts) in children, and 34 chains in
adults (HbH)
o (--/-a)
- Hydrops fetalis (Homozygous a-thal-1)
o Causes total absence of a-globin
synthesis
o Excess y globin forms tetramers called Hb
Barts, which has very high 02 affinity
▪ It delivers almost no O2 to
peripheral tissues leading to
asphyxia, edema, CHF, and death.

α - / ααα trait TRAIT HbA


α-/α- Mild/heterozygous HbA
α-/-- HbH HbH
--/-- Hb Hydrops fetalis Hb Barts

DIAGNOSIS OF THALASSEMIA SYNDROMES

- The diagnosis of B-thalassemia major is readily


made during childhood
o Hb electrophoresis shows inc. Hb A2,
HbF, or both. Little to no HbA is present
o B-thalassemia minor
▪ Hb electrophoresis shows inc/nor.
HbA2, HbF
o a-thalassemia trait
▪ HbA2 and HbF are normal, mild
hypochromia and microcytosis
BLOOD BANK o Elderly – expect ↓ Ab = ↓ reaction
potential
ABO BLOOD GROUP
ABO Antibodies
Blood Groups
IgM
• Major blood groups – usually use for blood
- “Naturally occurring form”
transfusion and organ transplantation
- COLD-reacting
o ABO (most important)
- BIND complement – activated by IgM & IgG
o Rh
- DO NOT CROSS placenta
• Minor blood groups
IgG and IgA
ABO Blood Group System
- “Immune forms” or secretory forms
• Discovered by Landsteiner, von Descatello and
- Results from foreign red cell stimulation by
Sturle
transfusion or pregnancy
o Drew blood from himself and 5
o Fetal Maternal Hemorrhage (FMH) –
colleagues
bleeding caused by labor
o Separated serum from the red cells
and mixed them up ABO Antigens
o Unknowingly performed the first ever
ward and reverse blood typing General characteristics

Landsteiner’s Experiment - Glycoproteins or glycolipids


Red Cells Sera from A Sera from B - Present in all organs of the body – blood
person (Anti- person (Anti- transfusion & organ transplantation
B) A) (histoblood group antigens)
O person (No NEG NEG - Phenotypic or physical expression affected by
Ag) several factors:
A (A Ag) NEG + o Age - Develop as early as 37th day of
B (B Ag) + NEG fetal life; Newborn red cells have only
AB (A and B + + 25 to 50% of the number of antigenic
Ag's) sites found on adult red cells
o Race – but some races do not
demonstrate ABO usually Bombay
Blood Typing:
blood group
• Forward – unknown Ag + known Ab o Genetic interaction – A1, A2
• Reverse – unknown Ab + known Ag o Disease states – acquired A, acquired B
- Inheritance
Landsteiner's Experiment o Alleles involved: A, B, O, H and Se
- Individuals have "naturally occurring" Blood Group Genes
antibodies in their serum directed against the A AA, AO (homozygous A,
missing antigens on the surface of their RBCs heterozygous A)
- Naturally occurring antibodies against the ABO B BB, BO (homozygous B,
blood group heterozygous B)
- Significance: once you’re exposed to wrong AB AB (heterozygous AB)
blood type there will be an immune reaction O OO (homozygous OO)
Dominant – AB blood groups
What We Know Today
Recessive – O blood groups
- ABO antibodies are not really "naturally
occurring". Their production is stimulated by - Formation
ubiquitous substances such as: o Precursor substance: paragloboside
o Bacteria ▪ Red cell surface
o Pollen • Type 2 (terminal
o Other substances galactose and N-
- Production of ABO antibodies starts at birth acetylglucosamine in
but titers are too low for detection until 3 to 6 beta 1 → 4 linkage) =
months of age. (postpartum) Glycolipid is formed.
- Antibody production peaks at 5 to 10 years but ▪ Body fluids and secretions.
decreases with age. • Type 1 (terminal
- Persons above 65 have low titers for detection. galactose and N-
acetylglucosamine in
beta 1 → 3 linkage) = Glycosyl Transferases
Glycoprotein is Ge Enzyme Nucle Sugar Anti
formed. ne otide gen
(Sugar
NOTES: Donor
)
Red cell surface – glycolipid
H α-2-L- GDP- L-fucose H
Body fluids and secretions – glycoprotein fucosyltransferase Fuc
A α-3-N- UDP- N- A
o Addition of Sugars acetylgalactosami GaINA acetyl-
▪ Glycosyltransferases nyltransferase C D-
▪ Type of glycosyltransferase galactos
depends on inherited alleles amine
B α-3-D- UDP- D- B
GEN GLYCOSYLTRANSFERASE IMMUNODOMIN galactosyltransfera Gal galactos
E ANT SUGAR se e
H α-2-L-fucosyltransferase L-fucose O None None None O
A α-3-N- N-acetyl-D- NOTES: O is none because it only has H
acetylgalactosaminyltrans galactosamine
ferase Glycosyl Transferases
B α-3-D- D-galactose
• GDP-Fuc: guanosine-diphosphate L-fucose
galactosyltransferase
• UDP-GalNAC: uridine diphosphate-N-acetyl-D-
galactose
NOTES: • UDP-Gal: uridine diphosphate galactose

Sugar A – Soluble A, B and H Antigens (present in secretions)

Sugar H – Secretion

- You need L-fucose for your A or B blood group - Secretor status (Sese or SeSe): α-2-L-fucosyl
sugars to bind. transferase expression in tissues related to
o e.g., Patient (A+) exocrine secretions
▪ in the RBC there should be a o Secretes A, B, H antigens in saliva,
present of L-fucose before it urine, mucus secretions. How? It’s a
binds to N-acetyl-D- histoblood group. In all of our
galactosamine secretory organs, there’s a present α-
▪ if the case is you do not have 2-L-fucosyl transferase. If you have a
L-fucose, whether A or B yung presence of L-fucose in your
na-iinherit mo, you will secretions/secretory organs, may mag
present as O blood group. bibind dito na A or B Ag resulting in
Why? Because there’s no secretions of either A, B, AB, O Ag in
binding antigen and therefore saliva, milk, mucus secretions, etc.
there’s no reaction to anti-
sera.
▪ You need L-fucose to bind A or
B

- Non-seceretors (sese) – heterozygous

Fluids in which A, B and H substances can be


detected in secretors

- Saliva, urine, bile, amniotic fluid, tears,


digestive juices, milk and pathologic fluids
(pericardial, peritoneal, pleural)
Differences Between Red Cell and Soluble A, B Aa, Ab, Ac and Ad Antigens
and Antigens
Red cell A, B Soluble A, B - Based on the four different forms of the H
and H antigens and H antigens antigen
Biomolecule Glycolipids Glycoproteins o All boils down to anti-H antisera. Is it
First sugar Glucose N-acetyl problematic in blood transfusion? NO.
galactosamine because A1 and A2 are usually the
Chain Type 2 Type 1 most often associated with weak
Gene FUT1 FUT2 reactions, delayed HTRS
- H antigen forms: H1, H2, H3 and H4
o H1 and H2 are unbranched straight
**bakit nagkakaroon ng subgroups? Dahil may chains
mahihinang reaction. o H3 and H4 are complex branched chains
ABO Subgroups o H1 and H2 can be converted to Aa and
Ab by both A1 and A2 enzymes
A Subgroups o H3 and H4 can only be converted to Ac
and Ad by A1 enzyme and very poorly
- Main subgroup (2 schemes)
by A2 enzyme
o A1 and A2
o So: A1: has Aa, Ab, Ac and Ad
o Aa, Ab, Ac and Ad
A2: has Aa, Ab, low Ac and no Ad
- Weak subgroups
o Occur in 1% of the population Reagents Used to Detect A Subgroups
o A3, Am, Ax, Ay, Aend and Ael
Anti-A (ASUL → BLUE)
A1 and A2
- Extracted from sera of type B persons
- First described by von Dungern in 1911 - Contains both Anti-A and Anti-A1
- Quantitative and qualitative differences:
o Concentration of α-3-N-acetyl- Adsorbed Anti-A
galactosaminyltransferase - Sera from type B persons are adsorbed with A 2
o Number of antigen sites on red cell cells
o Antibodies produced - Contains only Anti-A1
A1 o Tinatanggal mismo sa solution, ang
natitira Anti-A1
- A1 gene elicits production of high
concentrations of α-3-N-acetyl- Dolichos biflorus lectin/Anti-A1 lectin
galactosaminyltransferase - Seed extract that agglutinates A1 cells
o Resulting in ↑ concentration of - Similar to Anti-A1
GALNAC - Used to detect Anti-A1 patients
▪ e.g., Sa RBC mo marami kang
GALNAC A Subgroup Typing
- Production of 810,000 to 1,170,000 antigen
sites Anti-A (from B Adsorbed Anti-
sera) = BLUE A/Anti-A1
- Presence of A and A1 antigens
lectin =
- Production of Anti-H cold agglutinin
Dolichos
o Sa sobrang daming A Ag, hindi na biflorus
naeexpose sa sariling H Ag A1 + +
A2 A2 + NEG

- Less activity of α-3-N-acetyl-


galactosaminyltransferase than A1 Weak A Subgroups (page 131 – table 6-15)
- Production of 240,000 to 290,000 antigen sites
- Presence of only the A antigen
- Production of Anti-A1
o Merong presence ng H and A na
nakikita ng immune cells mo. Problem
is few A, kaya nagpoproduce ng Anti-
A1

A subgroup Ag Ab
A1 A, A1, *H Anti-H, anti-B
A2 A, H Anti-A1, anti-B
B Subgroups Hypogammaglobulinemia Weakened Anti-A and B
Agammaglobulinemia
Main subgroup Very young and old
populations
- α-3-D-galactosyl transferase:
Polysaccharides from E. Acquired B antigen
o Requires Mn2+ as cofactor
coli (Due to any
o Optimum pH 6.5 condition that increases
o Two types: pl’s 4.8 to 5.2 and 8.2 to permeability of
8.8 intestinal wall)
- Also based on the four different forms of the H Excessive amounts of Absence of antigens
antigen: BI, BII, BIII, and BIV (not usually used in blood group-specific during forward typing;
the lab) soluble substances (Due Patients appear to be
to gastric and blood type O
Weak subgroups (page 134 – table 6-16) pancreatic cancers –
because of increase
- B3, BM, BX, Bel
secretion)

NOTES:

For babies – only forward typing

Some E. coli produce enzyme responsible for


converting GALNAC to galactosamine. The
galactosamine cross-reacts with Anti-B

FORWARD REVERSE
AB Subgroups A – 4+ RA - O
B – 2+ RB – 4+
- Related to A subgroups D – 4+
- A1B and A2B ** THIS TABLE IS FOR ACQUIRED B ANTIGEN**

Bombay Phenotypes If there is an increase consumption in the secretion


therefore wala kang nacoconsume sa RBC antigens
- Lack A, B and H antigens (doesn’t have
anchoring body) Pancreatic cancers
- Production of anti-H that react at 37°C (IgG)
- ↑ secretory phases (pancreatic juice or gastric
- Two types:
o Classical Bombay (doesn’t produce H enzymes) = ↑ secretions of A, B, H Ag = ↓ RBC
Ag) carbohydrates = +/- absence of Ag
▪ hh FORWARD REVERSE
o Para-Bombay A–O RA – 2+
▪ Ah, Bh and ABh B–O RB – O
▪ Weak expression of A and B D – 4+
▪ Mutant H (FUT) gene results in **THIS TABLE IS FOR GASTRIC/PANCREATIC CANCERS**
low levels of H antigen
▪ Usually 1+, 2+

Anti-H QUESTIONS AND ANDSWERS PORTION:

- A1 and A1B Anti-H are cold agglutinins BTRH (for forward and reverse typing)
- Bombay Anti-H react strongly at 37°C
Specimen: EDTA/red top
- Reactivity: O > A2 > B > A2B > A1 > A1B
1. Centrifuge the blood to separate plasma and
NOTES:
RBCs.
O – has greatest H antigen and so on… 2. Prepare 6 tubes. Each labeled with A, B, D, RA,
RB, RBC.
Disease States and Conditions that Alter ABH 3. Get the RBC in the bottom to make 5% RBC
Antigens suspension, get 0.5mL RBC and dilute it with
Condition Alteration 4.5 mL NSS to have 5mL (estimate).
Leukemia Weakened A and B 4. Get 1 drop of RBC suspension and put it on
Conditions related to antigen expression forward typing (A, B, D) to know the Ag in the
stress hematopoiesis
patient’s blood.
(e.g., thalassemia)
5. Get patient’s plasma if EDTA and serum if red
Hodgkin's disease
top. Then put it on reverse typing (RA, RB).
6. Get anti-sera and put it on their respective
tubes.

Anti-A for tube A


Anti-B for tube B
Anti-D for tube D
7. Get a prepared known cell. These known cells
are blood type to know what antigen are they.
Get known A and put it on Reverse A (RA) and Example given (problem to solve):
known B on Reverse B (RB). What you need to
know what’s on RA and RB? To know what Ab 6 months old, female:
patient has.
FORWARD REVERSE
8. The known A and B that you’ve get will be A–O RA – O
centrifuge to 3,500 rpm for 30 seconds. B – 4+ RB – O
Centrifuge the tube A, B, D, RA, RB. D – 2+
9. The IgG that are present in the anti-sera are Blood Type (BT): B pos
going to be potentiate (pagsasama-samahin).
If they are in only one suspension, hindi sila Reverse is negative because the patient is only 6
gaanong magkakalapit, hindi magiging malakas months old, new born baby and it still doesn’t produce
yung reaction. antibodies.

37 years old, male:

FORWARD REVERSE
A–O RA – 4+
B–O RB – 4+
D – 4+ RO – 4+
Blood Type (BT): Bombay

Because it has Reverse O (RO) which has L-fucose (H


Ag). Nagkaroon ng reaction sa Reverse H, sa Reverse H
na demonstrate na mayroong anti-H production si
patient which only happens in A2 and Bombay. Walang
After centrifugation dislodged the RBC button. If the forward na A2 so it’s not A2. So kahit ano yung ma-
RBC button has reaction or none, naka dikit ito sa inherit na A or B Ag hindi ito mag dedemonstrate sa
basement ng tube. After that, grade their forward because walang anchoring (L-fucose).
agglutination reactions then dito na papasok yung 4+,
NOTES:
3+, etc.
- RO or anti-H is the same
- Why we do reverse typing?
o For confirmation if forward is right and
if possible of it having other Ab
present. For the safety of the patient.
- BTRH = A positive
o Rh Blood Group are seen in anti-D anti-
sera, Rh is composed of DCEce, D is the
major antigen of rhesus.

Blood Type: A pos


Blood Antigen Antibody (Ab)
Group (Ag)
(BG)
A A, H anti-B, A1 = anti-H, A2 = anti-A1
B B, H anti-A
AB A, B, H none
O H anti-A, anti-B
Bombay none anti-A, anti-B, anti-H
**anti-H → weak

FORWARD REVERSE
A–O RA – 4+
B–O RB – 4+
D–O
Blood Type (BT): O neg

For blood transfusion:

- Patient’s identification
- BTRH (both forward reverse is done) MT
- Cross-matching (I, II) → IS, LISS, AHG (37°C)
- Pre-transfusion
- Transfusion RN, MD
- Post-transfusion
BLOOD BANK subscripts written as superscripts (Rh0,
Rh1, Rh2, RhZ, rh, rh', rh”, rhy).
Rh Blood Group - Rules for longhand:
o Rh0 refers to “D”
Rh Antigens o Single prime (') refers to “C or c”
o Double prime (“) refers to “E or e”
- Nonglycosylated transmembrane proteins o If “r” precedes “h” (rh' or rh”), we are
- Started out with just 5 antigens (D, C, c, E and referring to the uppercase “C and E”
e) o If “h” precedes “r” (hr' or hr”), we are
- 51 antigen specificities (refer to Table 6-5 of referring to the lowercase “c and e”
Harmening) - Example:
- Immunogenicity: D > c > E > C > e o Dce converts to Rh0hr'hr”
- Genetics still unclear - Rules for shorthand:
o “R” denotes presence of “D”, “r”
Rh Antibodies
denotes absence.
- Production due to sensitization o “C” denoted by subscript “1 (one)” or
- IgG (subclasses 1 to 4) by a single prime (')
o “E” denoted by subscript “2 (two)” or
- Do not usually bind complement
by a double prime (“)
- Extravascular hemolysis
o When both “C and E” are uppercase,
- IgG1 and IgG3 subclasses are of greatest clinical
the subscript “z” or superscript “y” is
significance
used.
- React optimally at 37°C or after antiglobulin o When both “c and e” are lowercase,
testing the subscript “0” or no superscript is
- Can cross placenta used.
- IgA and transient IgM forms o Subscripts are used with uppercase
“R”, while superscripts and primes
Nomenclature
with lowercase “r”.
- 4 schemes: Fisher-Race, Wiener, Rosenfield - Example:
and ISBT o R1 converts to DCe

Fisher-Race (DCE) SUMMARY OF WIENER TERMINOLOGY


Antigen Longhand Shorthand Fisher-
- Theory: Race
o Antigens produced by three closely Rh0 Rh0hr'hr” R0 Dce
linked alleles (D, C/c and E/e) Rh1 Rh0rh'hr” R1 DCe
o One antigen per allele Rh2 Rh0hr'rh” R2 DcE
o Codominant expression RhZ Rh0 rh'rh” RZ DCE
- Antigen designations: D, d, C, c, E and e rh hr'hr” r dce
- Rules: rh' rh'hr” r' dCe
o Antigens have same designations as rh” hr'rh” r” dcE
genes. Genes are italicized. rhy rh'rh” ry dCE
o “d” refers to the absence of D antigen.
o “-” represents other missing antigens Rosenfield (Alpha/Numeric)
o Haplotypes are inherited from each
parent - Theory: No genetic basis. Simply states
- Example: presence or absence of Rh antigens.
o Genotype: DCe/dce - Rules:
o Phenotype (antigens produced): D, C, o Minus sign (-) before the number
c and e designates absence.
o If an antigen hasn't been tested for, its
Wiener (Rh-Hr) number will not appear.
o Designations:
- Theory: three antigens produced by single
 D: Rh1
gene (Rh0)
 C: Rh2
- Antigens: Rh0, Rh1, Rh2, Rhz, rh, rh', rh”, rhy
 E: Rh3
- Longhand and shorthand designations
 c: Rh4
- Rules for writing genes:  e: Rh5
o Antigens have same designations as o Examples:
genes. Genes are italicized, with  D +, C + E +, c -, e - converts
to Rh: 1, 2, 3, - 4, - 5
- Drawback: Other blood groups such as: Kell, - Limited amount of precursor 1 converted to
Duffy, Kidd, Lutheran, Sciana, etc. have precursor 2
similar nomenclature scheme. - Weak expression of CDE or LW antigens
- Individuals may produce antibodies to Rh
D. ISBT
antigens they lack
- Six digit numbers
Clinical Considerations
- 004 was assigned to Rh blood group system
(e.g., Rh1 → “D” → 004001) - Transfusion reactions
- Similar to Rosenfield, only difference: o Circulating Rh antibodies appear
o RH instead of Rh within 120 days of primary exposure
o Takes genetics into account. and 2 to 7 days after secondary
Characters are italicized, followed by exposure.
a space or an asterisk, and then o Unexplained fever, mild bilirubin
numbers of the antigens are separated elevation and decreased hemoglobin
by commas. and haptoglobin
- Example: Rh1 converts to DCe and RH 1,2,5 or o Positive DAT
RH*1,2,5 - HDN
o RhoGAM (IM injection)
Proposed Genetic Pathways
Hemolytic Disease of the Newborn (HDN)
- Fisher and Race theory
- Wiener theory - Destruction of fetal red blood cells by
- Currently accepted model maternal antibodies
o Two linked genes on chromosome 1 - Mother is stimulated to produce antibodies by
control Rh expression previous pregnancy or during 2nd or 3rd
 Codes for presence or absence trimester of pregnancy
of “D” - Two common types:
 Codes for Ce, cE, ce and CE o Rh HDN
o Codominant expression o ABO HDN
- Cause: Usually antibodies (IgG) to the Rh(D)
Mechanisms of Antigen Expression
antigen
- Couples usually affected:
o Rh negative female
o Rh positive male
- Second child is usually affected
- Prevention: Rh immune globulin (RhIg or
Rhogam)
Rh Subgroups Factors Affecting Immunization and Severity
- Rhnull - Antigenic exposure
- Rhmod - Host factors
Rhnull - Immunoglobulin class
- Antibody specificity
- Regulator/ Inhibitor - ABO group influence
o Genes: homozygous X0r/X0r
o Precursor 1 not converted to precursor Antigenic exposure
2
o No CDE or LW antigens - Fetomaternal hemmorhage
o Individuals may produce antibodies to o Pregnancy (Transplacental)
any of the Rh antigens o Delivery
- Amorphic - As little as 1ml of fetal RBC can already
o Genes: rr immunize mother
o Precursor 1 converted to precursor 2 Conditions that Promotes Fetomaternal
o Unaltered precursor 2 Hemmorhage
o Individuals may produce antibodies to
any of the Rh antigens - Amniocentesis
- Chorionic villus sampling
Rhmod
- Abortion
- Genes: homozygous XQr/XQr - Ectopic pregnancy
- Abdominal trauma
- Accidental or inadvertent transfusion Pathology: Bilirubin Elevation
- Greater than 40 weeks gestation
- Newborn liver unable to conjugate bilirubin
Host factors - This results in elevated indirect
(unconjugated) bilirubin
- Depends on complex genetic factors - Kernicterus – permanent brain damage
Immunoglobulin Class Diagnosis: Serologic and Clinical Tests
- IgG starts crossing the placenta during the - Serologic tests
second trimester o ABO and Rh typing
- IgG1 and IgG3 are more efficient in RBC o Antibody screen: 37ºC and potentiator
hemolysis (PEG or LISS)
- IgG half-life: 25 days o Treatment with dithiotreitol or 2-
mercaptoethanol destroys J-chain of
Antibodies that causes HDN
IgM; and differentiates IgG from IgM
Common: - Clinical tests
- Amniocentesis and cordocentesis
- Anti-D o Amniotic fluid is read
- Anti-D + C spectrophotometrically at 450nm
- Anti-D + E o Readings are plotted in a Liley graph
- Anti-C o Bilirubin causes a change in optical
- Anti-E density
- Anti-c
New Diagnostic Techniques:
- Anti-e
- Anti-K - Ultrasonography
- Doppler assessment of middle cerebral artery
Rare:
peak velocity
- Anti-Fya - Percutaneous umbilical blood sampling
- Anti-s - Allele-specific gene amplification studies on
- Anti-M fetal cells
- Anti-N - Intrauterine transfusion
- Anti-S Treatment
- Anti-JKa
- Intrauterine transfusion:
Antibody Specificity o Amniotic fluid change in OD450 results
- Anti-D is most potent are in high zone II or III
o Cordocentesis blood sample has
- Other Rh antibodies: Anti-E and anti-c causes
hemoglobin less than 10 g/dL
severe HDN
o Fetal hydrops is noted on ultrasound
- Other blood group antibodies: Anti-Kell is most
examination
potent
- Early delivery
ABO Group Influence - Phototherapy
- Newborn transfusions
- ABO incompatibility between mother and fetus
prevents mother from being sensitized. Newborn Transfusion:

Pathology - Purpose:
o Removal of bilirubin
- Erythroblastosis fetalis and hydrops fetalis o Removal of sensitized RBCs and
- Bilirubin elevation incompatible antibody
Pathology: Erythroblastosis Fetalis o Suppression of erythropoiesis
- Blood must be:
- Red cell destruction →Overworked bone o CMV-negative
marrow → Spleen and liver produces red cells o Hematocrit 70%
→ Hepatosplenomegaly → Hypoproteinemia
→ Hydrops fetalis (Cardiac failure with
edema)
Prevention: Rh Immune Globin

- Dose: Volume of fetomaternal hemorrhage


divided by 30
- Fetomaternal hemorrhage volume:

Number of fetal cells X Maternal blood volume


𝑁𝑢𝑚𝑏𝑒𝑟 𝑜𝑓 𝑚𝑎𝑡𝑒𝑟𝑛𝑎𝑙 𝑐𝑒𝑙𝑙𝑠

ABO vs. Rh HDN


Characteristics ABO Rh
First pregnancy YES RARE
Disease NO YES
predicted by
titers
IgG YES YES
Bilirubin at NORMAL ELEVATED
birth
Anemia at birth NO YES
Phototherapy YES YES
Exchange RARE SOMETIMES
transfusion
Intrauterine NONE SOMETIMES
transfusion
Spherocytosis YES RARE
MT-IMMUNO 1 (Immunology and Serology) LEC Self- Very good; no Very good;
non/self microbe-specific occasional failures
Innate Immunity discriminati self/non-self of discrimination
on patterns in host result in
Immunity autoimmune
 Vertebrates are protected by two systems of disease
immunity. Soluble Many antimicrobial Antibodies and
- Innate immunity components properties, cytokines
 First and second line of defense of blood proteins, and other
- Adaptive immunity mediators,
 Takes time but has memory including cytokines
 Third line of defense Major cell Phagocytes T cells, B cells,
 Innate immunity can be found in all multicellular types (monocytes, antigen-presenting
plants and animals macrophages, cells
- Response remain the same even after neutrophils,
encountering the same microorganism/foreign dendritic cells), NK
bodies cells, other
 Adaptive immunity evolved in jawed vertebrates leukocytes,
- Improves over time and acquires this response epithelial and
and throughout its lifetime endothelial cells
 Humans have both innate and adaptive immunity
Innate Immunity
Innate and Adaptive Immunity  Innate immune system
Attribute Innate immunity Adaptive - Physical/Anatomical barriers
(Non- immunity  Skin and mucous membranes – impermeable
specific/Natural) (Specific/Acquired as long as intact, contains appendages such as
) the hair which trap and prevent pathogens
Response Minutes/hours Days (because it from entering the system
time involves activation - Mechanical barriers
of B cells and T  Peristalsis, tearing of eyes, urinating and
helper cells); sweating, coughing and sneezing
antigen has to - Chemical barriers
encounter the B/T  Acidity of stomach, antimicrobial molecules
cell in secreted by sweat and oil glands
corresponding - Cellular barriers (Second line of defense)
receptors  Cells with sensitive receptors that can detect
Specificity For molecules and Highly-specific; microbial invaders
molecular patterns discriminates  Include normal indigenous microflora that
associated with between even minor compete with pathogens for nutrients and
pathogens; foreign differences in space
bodies are molecular structure  Antimicrobial proteins – inhibitory peptides
discovered through of microbial or provided by the body’s microflora (chemical barriers)
pattern recognition nonmicrobial - Psoriasin – produced by the skin (only works for
receptors molecules; gram-negative becteria)
discovered through  Antimicrobial activity to E. coli – help when
antigen-antibody skin is scratched or cut to prevent infection
interaction since we are exposed to E. coli in fecal
Genetic A limited number Highly diverse; a material
diversity of conserved, germ- very large number - Saliva, tears, and mucous membranes (contain IgA
line encoded of receptors arising and lysozyme) help to wash invaders away as well
receptors from genetic as contain antimicrobial peptides
recombination of  Normal flora – help to out-compete pathogens for
receptor genes in space and nutrients
each individual  Innate responses can be found in:
Memory Some (observed in Persistent memory, - Skin
responses invertebrates with faster response - Mouth
innate responses of greater - Airway
and mouse/human magnitude on - Lungs
NK cells) subsequent - Epithelial linings of the airway and lungs
exposure - Epithelial lining of alimentary canal

dane.
- Stomach o C4b will bind/form a complex with C2a,
- Large and small intestine producing C4b2a (C3 convertase of the
- Rectum classical pathway) which will split C3,
- Urogenital tract which will form C3a and C3b.
- Salivary, lacrimal, and mammary glands o C3b will bind to C4b2a to form C4b2a3b
(C5 convertase), which will activate C5,
Organ or Tissue Innate mechanisms which will split into C5a and C5b.
protecting o C5b binds to C6, C7, C8, and C9.
skin/epithelium Altogether, they form the membrane
Skin Antimicrobial peptides, attack complex (C5b789).
fatty acids in sebum  Alternative
Mouth and upper Enzymes, antimicrobial o C3 is a relatively fragile protein. It
alimentary canal peptides, sweeping of activates on its own, splitting into C3a and
surface by directional flow C3b.
of fluid towards the o Upon stabilization by Factor D, Factor B,
stomach and properdin, C3b becomes C3
Stomach Low pH, digestive enzymes, convertase. C3b activates a protein,
antimicrobial peptides, fluid Factor B, into Bb and Ba. The Bb
flow toward intestine component binds C3b so it can activate
Small intestine Digestive enzymes, more C3.
antimicrobial peptides, fluid o If C3b binds with C3 convertase (C3bBb),
flow toward intestine it becomes C5 convertase (C3bBbBb)
Large intestine Normal intestinal flora which activated C5.
compete with invading o C5 splits into C5a and C5b, and C5b binds
microbes, fluid/feces to C6, C7, C8, and C9. Altogether, they
expelled from rectum form the membrane attack complex
Airway and lungs Cilia sweep mucus outward, (C5b789).
coughing, sneezing expel  MB-lectin pathway
mucus, macrophages in o Same with classical pathway with one
alveoli of lungs exception which is the initiating factor.
 Pattern Recognition Receptors (PRRs) Instead of the antigen-antibody, this has
- In contrast, antibodies and T cell receptors the mannose-binding protein and the
recognize finer detail of molecular structure mannose on the surface of the bacteria.
- PRRs recognize Pathogen-Associated Molecular o Lectin is a protein that can bind
Patterns (PAMPs) that are found on a variety of carbohydrates. In this case, it is a
microbes, instead of antigens (which is for mannose-binding protein.
Adaptive response) o Mannose acts as a substitute for the
antigen, while mannose-binding protein
 Complement System
acts as substitute for the antibody.
- One part is a collection of proteins that form
 Complement proteins are labeled C1 to C9.
aggregates that punch holes in pathogen’s cell
Activation is achieved when the protein is
membrane causing lysis
cleaved into two fragments, a larger
- Include serum glycoproteins that promote uptake
fragment (B) and smaller fragment (A),
of pathogens by phagocytes (opsonization –
applicable for all proteins except C2.
enhancement of phagocytosis)
 The larger fragments (B) are responsible for
 Complement system components coat cells to
punching holes in the in the bacterial cell
be recognized by phagocytes
walls/membranes, except for C2, where the
 Chemotaxis – signal for directed migration of
larger fragment is designated A. On the other
WBCs
hand, the smaller fragments (A) are
 Anaphylatoxin – mediate or trigger
responsible for mediating the immune effects,
inflammation
(chemotaxis, anaphylatoxin etc.), except for
- Complement system ties innate and adaptive
C3b which acts as opsonin.
immunity
 C3a, C4a, and C5a are anaphylatoxins.
- Complement activation
 C3b is an opsonin.
 Classical
 C3a and C5a are chemoattractants.
o Upon binding of antigen-antibody
complex (immune complex), activation of  Innate initiation of adaptive response
C1 will follow. C1 will split into C1a and - When pathogens infect cells, intracellular bacteria,
C1b. Simultaneously; C1a will then parasites, and virus-infected cells produce
activate C4 into C4b and C4a, and C2 to cytokines and other microbials that kill
C2a and C2b. intracellular pathogens. They can already die at
this point. However, if this is still not enough,
dane.
virus-infected cells can be killed by NK cells, if it - Inflammatory response develops – various
(virus-infected cell) is coated with antibodies. cytokines and inflammatory mediators act on
- When an infected by intracellular pathogens, a cell endothelium of blood vessels
can secrete cytokines and chemokines which can  Increased expression of Cell Adhesion
initiate inflammatory response. Molecules (CAMs)
- For extracellular pathogens, it can result in  Cells, such as neutrophils, adhere to
activation of complement proteins, which will endothelium using these CAMs strongly
cause it to lyse (antimicrobial proteins and enough not to be swept away by flowing
peptides, MBL, CRP, complement proteins activate blood
complement pathways which induce lysis) or  Then they must penetrate the wall of the
produce opsonins. Once opsonized, this can be vessel to move into the tissue
phagocytosed by macrophages and neutrophils.  Neutrophil Extravasation
- If phagocytosed, phagocytes can produce - Rolling
cytokines and chemokines can activate T cells, or  Selectin-mucin interactions mediate rolling
at the same time, present the phagocytosed - Activation of chemoattractant stimulus
antigens to T-helper cells, which will then trigger  Chemokines/chemoattractants induce change
an adaptive immune response in integrins
- Arrest and adhesion
Inflammation  Integrins adhere firmly to ICAMs
 Its goal is to arrest the pathogen or to limit its spread - Transendothelial migration to the tissue
to allow for more focused innate immune response. (diapedesis)
 Upon arresting a pathogen, all WBCs will go to that
location to facilitate tissue repair. Antimicrobial Peptides
 Hallmarks  Isolated from humans, frogs, flies, nematodes, plants
- Swelling (tumor)  Range from 6-59 amino acids long
- Redness (rubor)  Good source in humans is the neutrophil
- Heat (calor)  Work by disrupting microbial membrane
- Pain (dolor) - How do they discriminate between microbial and
- Loss of function (functio laessa) host membrane?
 Within minutes of tissue injury: - Big area of research
- Vasodilation – blood vessels dilate, resulting to
rise of blood volume to area, which explains why C Reactive Protein
there is redness and heat.  Inflammation-associated protein
 Tissue damage causes a release of vasoactive  Recognizes ligands on surface of microbes
and chemotactic factors that trigger a local - Produced by liver in response to inflammation
increase in blood flow and capillary  Elevated levels are indicative of disease
permeability - Opsonin
- Vascular permeability increases – accumulation  Helps in phagocytosis
of fluid (plasma in the blood leaks from the blood - Activates complement-mediated attack
vessel to the extravascular space/tissues, causing
the swelling) Type I Interferons
 Edema  When interferons (kind of cytokine that has particular
 Permeable capillaries allow an influx of fluid effect on viruses) bind to the receptors in immune
(exudates) and cells cells, it induces the cell to produce anti-viral proteins
- Leukocytes adhere to endothelial cells and that
pass through walls of capillaries and tissues – - Blocks viral protein synthesis
extravasation - Degrades viral DNA
 Diapedesis/Extravasation – squeezing of - Blocks assembly of viral peptides.
WBCs through the gaps of endothelial cells
 Phagocytes migrate to the site of Pattern Recognition Receptors – Toll like Receptors
inflammation (chemotaxis)  1980s
 Phagocytes and antibacterial exudates - Toll in flies
destroy bacteria  Important in fly development
 Othereffects  1996
- Chronic inflammation – persisting inflammation - Toll in fruit flies
 Cancer  Mutation caused susceptibility to infection of
 Necrosis fungus
- Fibrosis – replacing of healthy tissue with scar
tissue
 Extravasation

dane.
 1997 (Janeway) - Opsonization
- Found out that Toll-like receptor activated  Fc receptors – antibody binds antigen with
expression of immune human genes Fab portion, Fc portion is free to bind with Fc
 Made of leucine-rich repeat sequences receptors on immune cells for ADCC,
phagocytosis, etc
Cell Types of Innate Immunity  Enhancement of phagocytosis
 Neutrophils  Phagocytes have receptors for Fc portion of
- Phagocytosis anitbody
- Reactive oxygen and nitrogen species - Complement fixation
- Antimicrobial peptides  Activation of complement by antigen-
 Macrophages antibody complex
- Phagocytosis - Antibody-dependent cell-mediated
- Inflammatory mediator cytotoxicity (ADCC)
- Antigen presentations  Cells that have cytotoxic potential (NK cells,
- Reactive oxygen and nitrogen species macrophages, etc) express receptors for Fc
- Cytokines portion of antibody
- Complement proteins  Antibodies bind antigens on the surface of
 Dendritic cells target cells
- Antigen presentation  NK cell CD16 Fc receptors recognize cell-
- Costimulatory signals bound antibodies
- Reactive oxygen species  Cross-linking of CD16 triggers degranulation
- Interferon into a lytic synapse
- Cytokines  Tumor cells die by apoptosis
 Natural killer cells (T-cytotoxic cells) - Degranulation
- Lysis of viral-infected cells  As seen in basophils during type 1
- Interferon hypersensitivity, occurs when antibodies
- Macrophage activation bound to WBCs via their Fc receptors, bind to
the same antigen
Signal Transduction Pathways  This cross-linkage will result in the same WBC
 Signal – microbial product releasing it granular contents (mostly
 Receptor – extracellular portion of TLR anaphylatoxins)
 Signal transductor – interactions of intracellular  Antibody isotypes mediate different effector
molecules—phosphorylation; signal transduction functions
pathway—promotes phosporylation of transcription - IgM
factors in nucleus  Fixing complement
 Effector mechanism – production of proteins, cell  Very good at grabbing and agglutinating
differentiation, inflammation, antigen-presentation, pathogens
etc. - IgG
 Fix complement, although not as efficiently as
Adaptive Immunity IgM
 Mediate ADCC
Components  Opsonin
- IgA
 Humoral – mediated by antibodies
 Neutralize toxins and pathogens (first line of
 Cell-mediated – mediated by T-cytotoxic cells
defense)
 In mucous membranes and secretions
Antibody-Mediated Effector Functions
- IgE
 Antibodies don’t directly kill pathogens by themselves.
 Inflammation, parasite
They are able to do so because of the following
 Interactions with basophils and anaphylatoxin
- Neutralization
histamine
 Occurs when antibodies coat a virus or other
antigen, preventing it from binding other cells
Cell Mediated Immunity
or tissue
 Important in detecting and eliminating cells that
- Agglutination
harbor intracellular pathogens
 Involves antibody binding antigen
 Antigen-specific
 Although in such cases, the antibody-antigen
- CD8+ and CD4+ T cells
complex is still bound to the cells, cross-link
with one another, causing cells or pathogens  Effector responses
to aggregate or agglutinate - Importance is evident when system is defective
 DiGeorge syndrome
o Born without thymus, lack T cell
requirement
dane.
o Can fight extracellular pathogens but have Vaccination
issues with intracellular pathogens (virus,
intracellular bacteria) Immunization/Vaccination
- Effector T cells express variety of effector  Immunity can be achieved by active or passive
molecules that are crucial in effectiveness of immunization
effector response - Active – long term production, immunologic
 Cytotoxic T cells memory, actual exposure
o CTLs or Tc cells, CD8+  Coming into contact with the foreign
o Class I MHC restricted substances
- Effector phase of CTLs  Vaccines
 CTL-mediated death - Passive – transfer of preformed antibodies
o Initiated by conjugate formation of CTL  Maternal antibodies to fetus (IgG to the fetus;
with infected cell IgA colostrum)
o Membrane attack, CTL dissociation  Antibody therapy for bites, immunodeficiency
o Target cell destruction either through  Temporary; antibodies will only last so long,
perforin (cell membrane disruption and but it can help someone recover from a
subsequent lysis) or Fas-FasL (binding of current infection
Fas receptor to Fas ligand) pathway  To achieve lifetime immunity, it must be
(induces apoptosis in target cell) through active immunization
 NK cells  There is a chance of side effects in a small number of
- Defend against viruses, other intracellular population
pathogens, and tumors - In the case with any treatment/drug
- Produce important cytokines - However, if the benefits to the population
- Lymphoid cells outweigh the risk of side effects, vaccines must be
 Share early lineage to T cells used to protect the majority of the population
 NK cells don’t develop exclusively in thymus - Herd immunity is important
- Killing of target cell is similar to that of CTL  There might be some people who can’t be
- Follow the opposing signals model vaccinated sue to cancer, allergies, etc
 Many different receptors for activation and  Just because you got the vaccine, it doesn’t
different inhibitory ones mean it worked on you
- NK receptors  Designing effective vaccine
 Lectin-like – actually bind proteins instead of - Protective immunity must be achieved
carbohydrates  Must pay attention to how the antigen
 Immunoglobulin-like activates the humoral and cell-mediated
- Balance between activation and inhibitory signals branches
allows NK to distinguish between self and non- - Must produce immunologic memory
self, very complicated  Vaccine that produces primary response but
- Activation and inhibition of NK cell is achieved fails to produce secondary response is not
depending which ligand effective
 NKT cells
- Cells that have characteristics common to CTLs Classification of Common Vaccines for Humans
and NK cells  Live, Attenuated Vaccines
- Considered as part of the innate immune system - Microorganisms can be attenuated so that they
- Role remains to be completely defined lose ability to cause significant disease
- Retain capacity for growth in host
Adaptive Immunity - Bacteria are grown for prolonged period in
 Adaptive immunity tends to be found in vertebrates adverse conditions; those that survive will not be
 However, do find innate immunity in non-vertebrates, suited to grow in better conditions in host
even plants - A virus might be grown in cell type that is not
- Sea squirt (chordate) – complement, TLRs normal host; accumulates mutations that might
- Fruit fly – TLRs, antimicrobial proteins weaken it
- Tomato – oxidative bursts, enzymes that digest - Measles, mumps, rubella vaccines
fungi, plant can isolate isolation by strengthening - Advantages
cell wall  Can grow in host therefore producing
immunologic memory with only a single
vaccination
 Produces memory T cells
 Good for distribution in third world countries

dane.
- Disadvantages o Freund’s incomplete adjuvant – used in
 Possibility that it will revert to virulent form research
o Polio – 1 in 2.4 million chance that this
will happen Preservatives in Vaccine Vials
 Complications  Thimerosal – preservative in vaccine vials (ethyl
o Measles vaccine – encephalitis mercury not methyl mercury which is the bad kind
o Out of 75 million patients between 1970 that builds up in fishes and is bad for humans)
and 1993, only 48 cases
 Danger from remaining unvaccinated and Hypersensitivity Reactions and Allergies
getting disease is much greater than
complications to these proven vaccines Hypersensitivity Reactions
 Inactivated or “Killed” Vaccines  Hypersensitivity – responding inappropriately to an
- Inactivation of pathogen by heat or chemical antigen
means  Inflammatory response can have deleterious effects
 Not capable of replication in host - Tissue injury
 Epitopes have to be maintained after killing - Disease
process - Death
- Often require boosters  May develop in course of humoral or cell-mediated
- Risks response
 Pathogen has to be grown in large numbers - Immediate hypersensitivity
prior to inactivation – individuals involved in  Anaphylactic
manufacturing are at risk  Antigen-antibody complexes
 Some of the pathogen may not be killed  Manifests in minutes
- Pertussis, typhoid, flu vaccine - Delayed type hypersensitivity
 Subunit Vaccines  May occur in days
- Purified macromolecules derived from pathogens  4 types of hypersensitive reaction
- Toxoids - Types I-III – humoral
 Some bacteria are pathogenic because of - Type IV – cell-mediated
exotoxins they produce
 Purify exotoxin, inactivate it with Type I (IgE–Mediated) Hypersensitivity
formaldehyde to form toxoid that can be used  Antigen induces cross-linking of IgE bound to mast
to immunize cells and basophils with release of vasoactive
 The immune system is then able to neutralize mediators
the toxin when an infection occurs  Typical manifestations
- Conjugate - System anaphylaxis
 When subunit vaccines are poor immune - Localized anaphylaxis such as hay fever, asthma,
stimulators, they can be coupled with strong hives, big bites food allergies, and eczema
immune activators  Induced by antigens referred to as allergens
 Polysaccharide on H. influenzae (not  Induces humoral responses but induces high secretion
immunogenic) with tetanus toxoid which is of IgE
highly immunogenic - Fc portion of IgE binds with Fc receptors on mast
o Results in immunity to Haemophilus cells and basophils
- Recombinant vector vaccines - Degranulation occurs
 Still in clinical testing  Common components
 Viruses and other microbes that replicate in - Allergens
cells but don’t cause disease can be used to  Atopy – hereditary predeposition to
deliver fragments of a pathogenic microbe for development of immediate hypersensitivity
vaccination reactions to common antigens
- DNA vaccines o Allows non-parasitic antigens to easily
 Tend to insert microbes’ DNA into host cells, induce IgE response
allowing host cells to manufacture microbial - IgE
proteins and antigens that will serve as the  Normally lowest of all antibody classes in
vaccine serum
 Adjuvants – enhances immune system  Half-life is 2-3 days but once bound to mast
response and time with the immunogen cells or basophils, can last for weeks
o Virosome – reconstituted virus envelope - Mast cells and basophils
without any genetic information (used in - IgE binding receptors
influenza and hep A vaccine)  High affinity
o AS04 – alum salt containing LPS (highly  Low affinity
recognized by TLR4)

dane.
o Atopic individuals have higher amount of  Categories of Type I Hypersensitivity
soluble IgE receptor that has been shown - Systemic anaphylaxis
to increase IgE production by B cells  Quick, can be fatal
 Common allergens associated with type I  Respiration labored, blood pressure drops,
hypersensitivity bronchiole constriction, edema, shock
Plant pollens Drugs  Epinephrine treats, relaxes smooth muscle
Rye grass Penicillin and increases cardiac output (prevents
Ragweed Sulfonamides vascular collapse)
Timothy grass Local anaesthetics - Localized hypersensitivity reactions
Birch tree Salicylates  Limited to tissue or organ
Foods Insect products  Types
Nuts Bee venom o Allergic rhinitis – “hay fever”
Seafood Wasp venom o Asthma
Eggs Ant venom o Atopic dermatitis – eczema
Peas, beans Cockroach calyx o Atopic urticaria - hives
- Food allergies – can cause local reactions or
Milk Dust mites
anaphylaxis
Other allergens
 Asthma
Animal hair and dander
- Inflammatory disease
Latex
- Induce expression of adhesion molecules on
Mold spores endothelial cells for eosinophils and neutrophils
 IgE cross-linkage initiates degranulation  Cause significant injury because of toxic
- Once cross-linkage of antigen has occurred, enzymes, cytokines
intracellular signaling result in mast cell  Notice sloughing of the pseudostratified
degranulation ciliates columnar epithelial cells lining the
- Cooperation among protein and lipid kinases, bronchiole
phosphatases, rearrangement of cytoskeleton  Clinical Methods to detect type 1
1. Allergen cross-linkage of cell-bounf IgE - Skin testing
2. Antibody cross-linkage of IgE  Histamine – negative control
3. Chemical cross-linkage of IgE  Feather – plane tree pollen
4. Cross-linkage of IgE receptors by anti-receptor  Cat – birch tree pollen
antibody  Dog – grass pollen
5. Enhanced Ca2+ influx by ionosphere that increases  Horse – daisy pollen
membrane permeability to Ca2+  Sheep wool – alternaria (mold)
 Granulocytes produce molecules responsible for - Checking serum level of IgE
Type 1 Hypersensitivity  Susceptibility to Type 1
- Primary mediators - Air pollution can increase allergies
 Preformed and stored in granules – ready to - Farm animals and bacteria exposure early in life
go can decrease allergies
 Examples: histamine, proteases, eosinophil  Hygiene hypothesis
chemotactic factor, heparin - Diet
- Secondary mediators - Genetics
 Synthesized after reaction is initiated  Control of Type I
 Examples: platelet-activating factor,
- Avoiding contact
leukotrienes, prostaglandins, bradykinins,
- Immunotherapy
some cytokines and chemokines  Subcutaneous injections of allergens –
- Histamine
desensitization
 Formed by decarboxylation of amino acid
o Causes shift to IgG production instead of
histidine
IgE
 Major component of granules  Monoclonal anti-human IgE
 Effects observed in minutes
- Drug therapies
 Contraction of smooth muscle (intestinal and
 Antihistamines
bronchial), increase permeability of venules,
 Antileukotienes
increased mucus secretion by goblet cells  Steroids
- Leukotrienes and Prostaglandins
 Drugs that enhance production of second
 Increased vascular permeability
messenger cAMP
 Smooth muscle contraction o Prevents degranulation of mast cells
 Mucus production
 Epinephrine and epinephrine agonists
- Cytokines and chemokines
(albuterol)

dane.
Type II (Antibody-Mediated Cytotoxic) Hypersensitivity  Large amounts of these complexes can lead to tissue
 Antibody directed against cell surface antigens damage
mediates cell destruction via complement activation
or ADCC Type IV (Cell-Mediated) Hypersensitivity
 Typical manifestations  Sensitized TH1 cells release cytokines that activate
- Blood transfusion reactions macrophages or TC cells that mediate direct cellular
- Erythroblastosis fetalis (hemolytic disease of the damage
newborn)  TH2 cells and CTLs mediate similar responses
- Autoimmune hemolytic anemia  Typical manifestations
 Transfusion reactions - Contact dermatitis
- Due to exposure to microorganisms in gut, - Tubercular lesions
individuals have antibodies to blood types not - Graft rejection
their own  Some subpopulations of TH cells encounter antigen,
- Antibody attaches to RBC and initiates secrete cytokines and induce localized inflammatory
complement system to lyse RBC response
- After lysis  Most cases are not detrimental
 Hemoglobin detected in plasma, starts to filter  Intracellular pathogens and contact antigens that
through kidneys and found in urine induce delayed-type (type IV) hypersensitivity
(hemoglobinuria) Intracellular bacteria Intracellular viruses
 Hemoglobin converted to bilirubin – toxic at Mycobacterium tuberculosis Herpes simplex virus
high levels Mycobacterium leprae Variola (smallpox)
 Fever, chills, blood clotting Brucella abortus Measles virus
 Hemolytic disease of newborn Listeria monocytogenes
- Rh+ fetus, Rh- mother Intracellular fungi Contact antigens
- IgG antibodies cross placenta
Pneumocystus carinii Picric chloride
 Some of these antibodies may be anti-Rh
Candida albicans Hair dyes
antibodies
Histoplasma capsulatum Nickel salts
o Can have severe consequences
Cryptococcus neoformans Poison ivy
 Antibodies against ABO blood groups produce
less consequences, can be easily treated Poison oak
- Rhogam shot Intracellular parasites
 Given to mother Leishmania sp.
 Anti-Rh antibodies bind to fetal cells that  Sensitization phase and effector phase of DTH
might have entered mother’s system during - Sensitization phase
birthing process, facilitates clearing before  Antigen-presenting cells: Macrophages
there is a B cell response o Langerhans cells
 DTH-mediating cells
Type III (Immune Complex-Mediated) Hypersensitivity o TH1 cells generally
 Antigen-antibody complexes deposited in various o CD8 cells occasionally
tissues induce complement activation and an ensuing - Effector phase
inflammatory response mediated by massive  TH1 secretions
infiltration of neutrophils o Cytokines – IFN-γ, TNF-β, IL-2, IL-3, GM-
 Typical manifestations CSF, MIF
- Localized Arthus reaction o Chemokines – IL-8/CXCL8, MCP-1/CCL2
- Generalized reactions such as serum sickness,  Effects of macrophage activation
necrotizing vasculitis, glomerulonephritis, o Class II MHC molecules
rheumatoid arthritis, and systemic lupus o TNF receptors
erythematosus o Oxygen radicals
- Pathogens o Nitric oxide
 Meningitis  Prolonged DTH can lead to formation of granuloma
 Hepatitis  Contact dermatitis
 Malaria
- Drugs Chronic Inflammation
 Sulfonamides  Caused by infections, continuing physical damage to
 Penicillin tissue, obesity, and autoimmunity
- Autoimmune
 Rheumatoid arthritis
 Lupus
 Complexing of antigen plus antibody facilitates
phagocytosis and clearing of antigen

dane.
Tolerance, Autoimmunity, and Transplants  Peripheral tolerance – kills lymphocytes in
secondary lymphoid tissue
Horror Autotoxicus - Also, life span of lymphocytes regulated by
 Failure of host’s humoral and cellular immune systems apoptosis
to distinguish self from non-self  CTLA-4 on Treg cells play a role in inhibiting activated
- Autoimmunity effector T cells
- Can result in tissue and organ damage, can be fatal  This keeps the effector T cell population at a
 Some organ-specific autoimmune diseases reasonable level
Disease Self-antigen Immune - If there is more antigen to deal with, then more
response effector cells will be activated
Addison’s disease Adrenal cells Auto-antibodies - If not, it brings the activated population down,
Autoimmune RBC membrane Auto-antibodies helping to prevent autoimmunity and unnecessary
hemolytic anemia proteins
inflammation
Goodpasture’s Renal and lung Auto-antibodies
syndrome basement  Some antigens can produce tolerance, termed
membranes tolerogens than immunogens
Graves’ disease Thyroid- Auto-antibodies - High dosages of antigen in host
stimulating (stimulating) - IV or oral introduction
hormone receptor - Absence of adjuvants
Hashimoto’s Thyroid proteins TH1 cells, auto- - Low levels of costimulators
thyroiditis and cells antibodies  CD28 will bind to B7 and provide activating
Idiopathic Platelet membrane Auto-antibodies signals; however, it was discovered that
thrombocytopenia proteins another receptor, CTLA-4 will bind to B7 and
purpura
inhibit
Insulin-dependent Pancreatic beta TH1 cells, auto-
diabetes mellitus cells antibodies
Myasthemia gravis Acetylcholine Auto-antibodies
Organ-Specific Autoimmune Diseases
receptors (blocking)  Target antigen specific to organ or gland
Myocardial infarction Heart Auto-antibodies  Cellular lysis and chronic inflammation that can
Pernicious anemia Gastric parietal Auto-antibody damage organ
cells; intrinsic  Hashimoto’s thyroiditis
factor - Mainly middle-aged women
Poststreptococcal Kidney Antigen-antibody - Target is thyroid antigens
glomerulonephritis complexes - Goiter can form
Spontaneous Sperm Auto-antibodies
- Hypothyroidism – decrease
infertility
 Autoimmune anemias
 Systemic Autoimmune Diseases
- Pernicious anemia
Disease Self-antigen Immune  Antibody against membrane bound intestinal
response protein that uptakes B12 needed for
Ankylosing Vertebrae TH1 cells and TC
hematopoiesis
spondylitis cells, auto-
antibodies - Hemolytic anemia
Multiple sclerosis Brain or white Auto-antibodies,  Antibodies to red-blood cell antigens
matter immune complexes  Type II hypersensitivity
Rheumatoid Connective tissue, Auto-antibodies - Drug-induced anemia
arthritis IgG  Goodpasture’s Syndrome
Sclerodoma Nuclei, heart, lungs, Auto-antibodies - Antibodies against basement membranes in
gastrointestinal glomeruli and alveoli
tract, kidney - Leads to kidney damage and pulmonary
Sjogren’s syndrome Salivary gland, Auto-antibodies hemorrhage
liver, kidney,
- Type II hypersensitivity
thyroid
Systemic lupus DNA, nuclear Auto-antibodies,  Insulin-Dependent Diabetes Mellitus (Type 1)
erythematosus protein, RBC and immune complexes - Antibodies against beta cells that produce insulin
(SLE) platelet - Insulin is needed by cells to uptake glucose
membranes needed for cellular respiration
- Type IV hypersensitivity – cell-mediated
Tolerance destruction
 Number of mechanisms are in place to protect  In some autoimmune diseases, antibodies act as
individual from self-reactive lymphocytes antagonists
 Central tolerance – deleting T or B clones before - Bind inappropriately to receptor, resulting in
maturity if they have receptors that recognize self- overproduction
antigens with great affinity

dane.
 For example, upregulating a hormonal AIDS and other Immunodeficiencies
response without the presence of that
hormone Autoimmunity VS Immunodeficiency
 Grave’s Disease – auto-antibodies bind to  Autoimmunity – system attacks host cells and tissues
receptor for thyroid stimulating hormone  Immunodefiency – system fails to protect
resulting in over-stimulation of thyroid - Primary immunodeficiency
o Type II hypersensitivity  Genetic or developmental defect
 Myasthenia gravis – antibodies bind - Secondary immunodeficiency – acquired
acetylcholine receptors on motor end plate of
muscles – progressively weakened skeletal Primary Immunodeficiencies (Lymphoid
muscles Immunodeficiencies)
o Type II hypersensitivity  Combined – effects both B and T cells
 B-cell immunodeficiency
Systemic Autoimmune Diseases - Range from absence of B cells, plasma cells,
 Response is directed toward wide range of target immunoglobulins to absence of only certain
antigens classes of antibodies
 Systemic Lupus Erythematosus - Subject to bacterial infections but do well against
- Typically middle-aged women viral since T-cell branch is ok
- Fever, weakness, arthritis, skin rash, kidney  T-cell immunodeficiency
problems - Can affect both humoral and cell-mediated
- Produce autoantibodies to DNA, histones,  Primary immunodeficiencies are often detected early
platelets, leukocytes, clotting factors in life
- Excessive complement activation
- Type III hypersensitivity – immune complexes Primary Immunodeficiencies – Combined
 Multiple sclerosis Immunodeficiencies (CD)
- Numbness, paralysis, vision loss  Severe combined immunodeficiency (SCID)
- Inflammatory lesions in myelin sheath caused by - Deficiency in RAG1, RAG2, Artemis, or DNA-PK
T cells  No TCR or Ig gene rearrangement
- Epidemiology  Inheritance mode – AR
 Frequent in African-American and Hispanic - Reticular dysgenesis – defect in mitochondrial
women enzyme AK2 required for hematopoiesis
 Environmental components as well as genetic  Block in differentiation of hematopoietic stem
components cells into myeloid and lymphoid lineages
 Rheumatoid arthritis  Inheritance mode – AR
- Chronic inflammation of joints - ADA and PNP deficiency
- Produce auto-Abs that bind Fc portion of IgG  Toxic purine metabolites in lymphoid
circulating in blood that creates immune progenitor cells. Leading to loss of B, T, and
complexes NK cells
- Type III hypersensitivity  Inheritance mode – AR (ADA), XL (PNP)
 Proposed mechanisms for induction of - Common γ-chain deficiency, JAK3 deficiency,
autoimmunity IL-7Rα deficiency
- Release of sequestered antigens  Defective signals from IL-2, -4, -7, -9, -15, -21
 Blood-brain barrier, sperm released into lead to blocks in development of T and NK
tissues during vasectomy cells
- Molecular mimicry  Inheritance mode – AR
- Inappropriate expression of Class II MHC - CD3δ, ε, or ζ deficiency
 Non-antigen presenting cells will for some  Defective signals from TCR block T-cell
reason express Class II MHC development
o Can be caused by viral infection  Inheritance mode – AR
 This allows them to present self antigen to T  Leaky SCID
helper cells – leads to inappropriate reaction - Hypomorphic mutations in proteins required for
V(D)J gene rearrangements, AK2, common γ
Treatment chains
 Immunosuppressive drugs - T cells present but with reduced repertoire
 Removal of thymus (for example, with myasthenia - B cells normal or decreased
gravis) - Igs reduced but may have increased IgE
 Plasmapheresis – removing plasma and then returning - May include oligoclonal T cells and autoimmunity
RBCs (removes extra immune complexes) (Omenn syndrome)
 Treating the inflammation - Inheritance mode – AR, AD, or XL
 Antigen given orally can induce tolerance
dane.
 Bare-lymphocyte syndrome (BLS): MHC - Inheritance mode – XL, AR
deficiencies  Chediak-Higashi syndrome
- Class I – β2-microglobulin, TAP peptide - Defective lysosomal trafficking regulator protein
transporter, tapasin (LYST)
 No/reduced MHC class I expression, reducing - Decreased CTL- and NK-mediated cytolytic
number of CD8+ T cells activity
- Class II – defect in transcription factors activating - Inheritance mode – XL, AR
MHC II gene expression  Leukocyte adhesion deficiencies
 No MHC class II molecules, reducing numbers - Defective integrins
of CD4+ T cells - Leukocyte extravasation and chemotaxis
- Inheritance mode – AR - Inheritance mode – AR
 Wiskott-Aldrich syndrome (WAS)  Selective NK-cell deficiencies
- Cytoskeletal protein (WASP) - Mutations in STAT5b, GATA2, MCM4, and RTEL1
- Defective T cells and platelets genes
- Inheritance mode – AR - Absent or decreased total NK cells or NK subsets
 Mendelian susceptibility to microbacterial - Increased susceptibility to viral infections
diseases (MSMD) - Inheritance mode – AR
- IFN-γR, IL-12, IL-12R, STAT1
- Impaired immunity to mycobacteria Primary Immunodeficiencies – Immune Regulation
- Inheritance mode – AR or AD Deficiencies
 DiGeorge syndrome  Autoimmune polyendocrinopathy with candidiasis
- Thymic aplasia and ectodermal dystrophy (APECED)
- Decreased number of T cells - AIRE defect
- Inheritance mode – AC - Reduced thymic negative selection and T reg
 Hyper-IgM syndrome formation
- Defective CD40 ligand or CD40 - Autoimmunity
- Elevated IgM due to loss of isotypes other than - Inheritance mode – AR
IgM; defective APC activation leading to reduced  Immune dysregulation, polyendocrinopathy,
T-cell responses to intracellular pathogens enteropathy, X-linked (IPEX) syndrome
- Inheritance mode – XL, AR - FoxP3 defect
- Absence of Tcreg cells
Primary Immunodeficiencies – Agammaglobulinemias - Autoimmunity
 X-linked (Bruton’s) agammaglobulinemia - Inheritance mode – XL
- Defective Bruton’s tyrosine kinase (Btk)
- Block in B cell development; no mature B cells Warning Signs of Primary Immunodeficiency
- Inheritance mode – XL  4 or more new ear infections within 1 year
 Reduction in one or more immunoglobulin  2 or more serious sinus infections within 1 year
isotypes  2 or more months on antibiotics with little effect
- Defective activation-induced cytidine deaminase  2 or more pneumonias within 1 year
(AID) or B cell surface proteins  Failure of an infant to gain weight or grow normally
- No/low IgG, IgA, IgE – various effects on IgM, IgG,  Recurrent, deep skin or organ abscesses
or IgA  Persistent thrust in mouth or fungal infection on skin
- Elevated or normal IgM  Need for intravenous antibiotics including septicemia
- No APC or T-cell deficiency  A family history of PI
- Inheritance mode – AR
 Common variable immunodeficiency Combined Immunodeficiencies
- Complex/unknown defect  Severe combined immunodeficiency (SCID)
- Low IgG, IgA; variable IgM - Low number of circulating lymphocytes
- Inheritance mode – complex - Non-proliferating T cells
 Selective IgA deficiency - Thymus doesn’t develop
- Complex/unknown defect - Usually fatal early years of life
- Low or no IgA  Infant will have viral and fungal infections
- Inheritance mode – complex  Bacteria don’t show up until later because of
placental transfer of antibodies from mother
Primary Immunodeficiencies – Innate Immune  Chronic diarrhea, pneumonia, lesions
Deficiencies - Many genetic defects can contribute to SCID
 Chronic granulomatous disease  Usually defects that target the early steps of
- Mutations in phagosome NADPH oxidase subunits hematopoiesis
- No ROS or RNS for killing of phagocytosed  MHC defects
pathogens - Sympyoms can resemble SCID
dane.
- Bare-lymphocyte syndrome  Defect limits recruitment of cells into areas of
- Failure to train T cells correctly, it affects both B inflammation
and T cells  Chediak-Higashi syndrome
 Thymus - Autosomal recessive disease
- DiGeorge syndrome – decreased or absent - Phagocytes don’t have ability to kill bacteria
thymus  Issue with lysosome activity
 Results from deletion of region on  Interferon-Gamma-Receptor defect
chromosome 22 in developing embryo, - Autosomal recessive trait – results from
developmental anomaly inbreeding
 Lowered T cell numbers, results in B cells not - Defect in receptor for IFN-γ and subsequent
producing sufficient antibodies pathways
 Wiskott-Aldrich syndrome (WAS)  Patients suffer from infection with
- X-linked disorder mycobacterium, showing importance of this
 Results in issues with cytoskeleton receptor in fighting mycobacterium
components in hematopoietic cells
- Initially B and T cell numbers are normal but will Myeloid Immunodeficiencies
decrease with age  Reduction in neutrophil count
- Treated with passive antibodies or stem cell - Low concentration – granulocytopenia or
transfer neutropenia
- Can result in fatal infection or lymphoid - Congenital neutropenia – frequent bacterial
malignancy infections
 X-linked Hyper-IgM syndrome - Acquired neutropenia
- Deficiency of IgG, IgE, IgA but elevated levels of  Certain drug or chemotherapy can cause this
IgM  Autoimmune disorder – destruction of
- Defect in T cell surface marker CD40L neutrophils
 This is needed for interaction between TH and  Complement defiencies
B cell for class switching for T-dependent - Fairly common
antigens - Mostly associated with bacterial infections or
 T independent antigens are not effected immune-complex diseases
therefore there is production of IgM
 Hyper-IgE syndrome (Job syndrome) Treatments for Immunodeficiencies
- Autosomal dominant  Replacement of missing protein
- Skin abscesses, pneumonia, eczema, facial - Administering immunoglobulin
abnormalities - Express genes in vitro (in bacteria) for cytokines
- High number of eosinophils and IgE but don’t  Replacement of missing cell type
show increased allergies - Bone marrow transplantation
 Replacement of missing or defective gene
B cell Immunodeficiencies - Gene therapy
 X-linked agammaglobulinemia  Still far out from clinical use
- B cell defect  Can it be corrected in vitro and then introduce
 Defect in kinase that keeps B cells in pre-B cells back into patient
stage with H chains rearranged but L chains
not Acquired Immunodeficiencies
- Low levels of IgG and absence of other classes  No genetic component
- Recurrent bacterial infections  Examples
 Common variable immunodeficiency (CVI) - Hypogammaglobulinemia – unknown cause,
- Low levels of immunoglobulin – different from genetic condition
hypogammaglobulinemia  AIDS caused by HIV
- Manifests later in life  HIV
- Undefined genetic component - Retrovirus (Lentivirus genus)
 Selective deficiencies of immunoglobulin classes - Viral envelope derives from host
- IgA deficiency is most common  Recognizes CD4 antigen on T cell
 Recurrent respiratory and urinary tract - Recognizes CD4 antigen on T cell
infections, intestinal problems - HIV-1 – main causative agent for AIDS
 Many different subtypes
Innate Immunodefiencies  Probably evolved from SIV and transferred to
 Leukocyte adhesion deficiency (LAD) humans during the “bushmeat trade” in early
- Integrin proteins needed for adhesion and cellular 1900s
interaction - HIV-2 – cousin to HIV-1 but disease progresses
slowly, if at all
dane.
 HIV Treatment
- Chemokine receptor antagonists
- Inhibit fusion
- Inhibit reverse transcription
- Inhibit integrase
- Inhibit protease
 AIDS does not fit the paradigm for classic vaccine
development
- Classic vaccines mimic natural immunity against
reinfection generally seen in individuals
recovered from infection; there are no recovered
AIDS patients
- Most vaccines protect against disease, not against
infection; HIV infection may remain latent for long
periods before causing AIDS
- Most vaccines protect for years against viruses
that change very little over time; HIV-1 mutates at
a rapid rate and efficiently selects mutant forms
that evade immunity
- Most effective vaccines are whole killed or live
attenuated organisms; killed HIV-1 does not
remain antigenicity, and the use of a live
retrovirus vaccine raises safety issues
- Most vaccines protect against infections that are
infrequently encountered; HIV may be
encountered daily by individuals at high risk
- Most vaccines protect against infections through
mucosal surfaces of the respiratory or
gastrointestinal tract; the great majority of HIV
infection is through the genital tract
- Most vaccines are tested for safety and efficacy in
an animal model before trials with human
volunteers; there is no suitable animal model for
HIV/AIDS at present

dane.
MT-IMMUNO 1 (Immunology and Serology) LEC - Immunogen – a substance capable of eliciting an
immune response
Introduction and Overview - All immunogens are antigens but not all antigens
are immunogens (i.e., haptens, incomplete
Immunology antigen incapable of eliciting immune response)
 Study of the immune system or immunity - Epitope – portion of the antigen that is recognized
 Immunity by an antibody or T cell receptor
- “Immunis” = exempt (Latin)
Systems of Immunity
History  Innate immunity (Non-specific)
 Discipline of immunology grew out of observation that - 1st and 2nd lines of defense
individuals who recovered from infectious diseases  Barriers that protect host (e.g., skin, acidity
were protected from disease of stomach, lysozymes in fluids)
 15th Century  Phagocytic cells
- Chinese and Turks tried to prevent smallpox  Antimicrobial peptides (e.g., interferons,
 Dried crust from pastules were inhaled or complements)
inserted into small cuts  Temperature
 1718 - Molecular and cellular mechanisms deployed
- Lady Montagu had that technique done on her before an infection
children - Distinguishes between self and pathogens but not
 1798 specialized to distinguish small differences in the
- Edward Jenner foreign particles
 Noticed that milkmaids that contracted - Less specific
cowpox were immune to smallpox  Adaptive immunity (Acquired and Specific)
 Inoculated a small boy with fluid from cowpox - Develops in response to infection
pustule - Adapts to recognize, eliminate, and remember
 Intentionally infected the boy with small pox – pathogen
the child did not develop smallpox - Highly specific
 1881 - 3rd line of defense
- Louis Pasteur  Antigenic specificity
 Vaccinated sheep with heat-attenuated  Diversity
anthrax  Immunologic memory
 Infected sheep with virulent strain of anthrax  Self-, non-self recognition
– the sheep did not develop anthrax - Effective adaptive immune response involves two
 1883 groups of cells
- Metchnikoff demonstrated that certain WBCs  Lymphocytes
were able to phagocytize microorganisms o B cells
 1901  Mature in bone marrow
- Von Behring and Kitasato demonstrated that  Antigen binding receptor: antibody
serum (noncellular component of blood) from made of glycoproteins
animals immunized to diphtheria could transfer  Glycoprotein structure
that immunity to non-immunized animals  Heavy chains – 2 identical
 1977 polypeptides
- Last naturally acquired case of smallpox  Light chains – 2 shorter identical
polypeptides
 In industrialized nations, measles, mumps, whooping
cough, tetanus, polio, and diphtheria are extremely o T cells
 Arise in bone marrow but mature in
rare or nonexistent
- This is due to vaccine thymus
- Prevents death, paralysis, deafness, blindness,  Two well defined subpopulations of
T cells
mental retardation
 T helper cells
Immune System  T cytotoxic cells
 T cells recognize antigen presented in
 Evolved to protect multicellular organisms from
MHC molecule
pathogens
 MHC – major histocompatibility
 Does this by two related activities – recognition and
complex
response
 MHC Class I – found on all of our
 Definition of Terms
nucleated cells (cytotoxic T cells
- Pathogen – something that causes disease
recognize this)
- Antigen – any foreign substance that binds
 MHC Class II – found on antigen
specifically to an antibody or T cell receptor
presenting cells (B cells, dendritic
dane.
cell and macrophages). Helper T - Myeloid progenitor cell
cells recognize this  Progenitor cells have lost ability for self renewal and
o Cytokines secreted by TH cells can are committed to particular cell lineage
activate phagocytic cells  Most complex stem cell system in the mammalian
o TC cells can kill altered self-cells body
 Cells infected by viruses  Regulated at gene level
 Tumor cells - Transcription factors play important roles in
 Antigen presenting cells hematopoiesis
o Phagocytosis of enemy cell (antigen) - Studies using knockout mice:
o Fusion of lysosome and phagosome  Gene inaction – if RBC or a particular WBC
o Enzymes start to degrade enemy cell fails to develop, it is concluded that protein
o Enemy cell broken into small fragments was involved in development of that cells
o Fragments of antigen presented on APC  Ikaros – transcription factor needed for
surface choice between B and T cell development
o Leftover fragments released by exocytosis  Notch – transcription factor needed for choice
between B and T cell development
Antibodies  Notch signaling is evolutionary conserved
 Antigen coated by antibody is eliminated in several pathway in multicellular organisms that
ways regulates cell fate determination during
- Can cross-link several antigens, making it easier to development and maintains adult cell
be ingested by phagocytic cells homeostasis
- Activate complement system resulting in lysis of
microorganism Hematopoietic Homeostasis
 Erythrocyte
Primary VS Secondary Immune Response - Average life span – 120 days
 Initial encounter with antigen causes primary  Leukocytes
response - Life spans from 1 day to 20-30 years
 Later contact with antigen will result in more rapid  Apoptosis – programmed cell death
response – secondary response
Cells of the Immune System
Immune Dysfunction  Myeloid lineage
 Allergies and Asthma - Monocytes that become macrophages (innate)
 Graft rejection - Dendritic cells (innate)
 AIDS - Granulocytes (innate) – neutrophils, eosinophils,
 Immunodeficiency basophils, mast cells
- Megakaryocytes (platelets) and erythrocytes
Cells and Organs of the Immune System  Lymphoid lineage – lymphocytes
- 20-40% of WBC
Stem Cells - 3 populations
 Ability to “self-renew” and to differentiate into  B cells (adaptive)
different cell types  T cells (adaptive)
 Embryonic stem cells  NK cells (innate)
- Capacity to generate almost all cell types
- Pluripotent Myeloid Lineage Cells
 Adult stem cells  First responders
- Capacity to give rise to the cell types that specify a  Mononuclear phagocytes
particular tissue - Monocytes circulate in blood and then migrate
- Multipotent into tissue and differentiate into specific
macropahage
Hematopoiesis - Activated macrophages are more effective than
 All blood cells (RBC and WBC) arise from resting ones
Hematopoietic Stem Cells (HSC)  Macrophages
 Study of these cells s difficult - Most tissue-resident macrophages arise early in
- Scarce embryonic development and have the ability to
- Difficult to grow in vitro continue to self-renew (multipotent)
- Isolation requires monoclonal antibodies and flow Tissue Name Tissue-specific
cytometry function
 Early in hematopoiesis, stem cell differentiates to Brain Microglia Neural circuit
either development
- Lymphoid progenitor cell (synaptic
dane.
pruning) - Antigen Presentation by Macrophage
Lung Alveolar Remove
macrophage pollutants and
microbes, clear
surfactants
Liver Kupffer cell Scavengr RBCs,
clear part
Kidney Resident kidney Regulate
macrophage inflammatory
responses to
antigen filtered  Dendritic Cells
from blood - Long membranous extensions, look like dendrites
Skin Langerhans cell Skin immunity on nerve cells
and tolerance - Antigen presentation – take a “snapshot” of what
Spleen Red pulp Scavenge RBCs, is happening in the tissues and carry this image to
macrophage recycle iron the lymph node
Peritoneal Peritoneal Maintain IgA - Four major groups
cavity cavity production by B-  Langerhans
macrophage 1 B cells  Interstitial
Intestine Lamina propria Gut immunity  Monocyte-derived
macrophage Regulate  Plasmatocytoid-derived
Intestinal peristalsis - Follicular dendritic cells
muscularis  Involved with B cell maturation
macrophage  Granulocytes
Bone marrow Bone marrow Maintain niche - Neutrophils
macrophage for blood cell  Multi-lobes nucleus, light colored granules
development,  First to arrive at site of inflammation
clear  High numbers is first indication of infection
neutrophils  Phagocytic
Lymph node Subcapsular Trap antigen  Generate antimicrobial agents
sinus particles  Very short-lived when compared to
macrophge macrophages
Heart Cardiac Clear dying  Come out of blood vessels ready to kill, would
macrophage heart cells cause too much damage to tissues if long-lived
- Eosinophils
- Includes:
 Phagocytic
 Intestinal macrophages in gut
 Play a role against parasitic organisms
 Alveolar macrophages in lung
- Basophils
 Histiocytes in connective tissue
 Non-phagocytic
 Kupffer cells in the liver
 Play a role in allergic reactions
 Mesangial cells in the kidney
- Mast cells
 Microglial cells in the brain
 Play important role in development of
 Osteoclasts in bone
allergies
- Monocyte VS Macrophage
 Similar to basophil cells

Lymphoid Lineage Cells


 B cells and T cells
- Adaptive immunity
- Small cells
- Those that have not interacted with antigen are
called naïve
 Interactions with antigen – proliferation into
effector cells (i.e. plasma cells) and memory
cells
 Natural killer cells – innate immunity
 Cluster of differentiation (CD) Markers – identifies
certain cell types

dane.
 Subpopulations
o T helper (TH)
o T cytotoxic (TC)
o T regulatory (Treg)
o Recent research shows there are even
more subpopulations
 T helper cells
 Lymphocytes o CD4 glycoprotein
- Once they have encountered antigen to which they o Recognizes antigen presented in MHC
are specific for, they will proliferate, making an Class II
army of B and T cells that are specific for an o “Help” activation of B cells, TC cells,
antigen macrophages in immune response
- Plasma or effector cells  T cytotoxic cells
 Plasma B cells – actively secrete antibody o CD8 glycoprotein
against the antigen o Recognizes antigen presented in MHC
 Effector T cells – actively search cells Class 1
presenting the antigen they are specific for o Eliminates infected cells or cancerous
- Memory B and T cells – last for decades, cells
watching for that same antigen to come back  T regulatory cells
 What causes a cell to become a plasma or o CD4 and CD25 glycoproteins
memory cell after activation is not yet known o Help suppress the immune system after
- Lymphocyte activation cycle (B and T cells) its been upregulated, after the infection is
cleared
 NK cells
o Innate immune response
o Large, granular
o Recognize tumor or virus-infected cells
o Antibody-dependent cellular cytotoxicity
(ADCC) or “kiss of death.”

Lymphoid Tissues and Organs


 Tissues and organs of the Immune System (AKA
Lymphoid)
 Primary
- Thymus
 T cell development and maturation
 Bilobed organ above heart
- B Lymphocytes (B cells)  Surrounded by capsule and divided into
 Site of maturation lobules (outer cortex; inner medulla)
o Bursa of Fabricius in birds  Network of epithelial cells, dendritic cells, and
o Bone marrow in mammals macrophages
 Display membrane-bound immunoglobulin  Thymus will induce death of those cells that:
(antibody) o Can’t recognize self-MHC molecules
 Once antigen is encountered o Those that interact with MHC molecules
o Differentiation too strongly
o Plasma cells – antibody can be secreted,  Function decreases with age
die within 1-2 weeks - Bone marrow
o Memory B cells – same-membrane  Lymphocytes arise there, but T cells go to
bound antibody as parent B cell, longer thymus to mature
life span  B cells mature here
- T Lymphocytes (T cells)  90% of plasma IgA comes from B cells in the
 Site of maturation - thymus bone marrow
 T cell receptor - Place of maturation of lymphocytes
o Only recognize antigen that is bound to - Lymphatic system
cell membrane proteins called major  Interstitial fluid (the portion that doesn’t
histocompatibility complex (MHC) enter venous system) is returned to
 Once antigen bound to MHC is encountered, it circulatory system by lymphatic vessels
differentiates into  Largest lymphatic vessel – Thoracic duct
o Effector T cells o Enters left subclavian vein
o Memory T cells

dane.
o Lymph from right arm and right side of  Innate immune cells – use pattern
head enters through right lymphatic duct, recognition receptors (PRRs)
drains into right subclavian  Lots of immune cells will express cytokine
 Antigen is carried by lymph to lymph nodes receptors
 Secondary o Cytokines are molecules that will elicit a
- Lymph nodes, spleen, mucosa-associated response in the immune cell
lymphoid tissues such as gut-associated lymphoid  Receptor-Ligand Interaction
tissues - Multiple noncovalent bonds between receptor and
- Provide venue for mature lymphocytes to interact ligand
with antigen  Hydrogen bonds
- Lymph nodes  Ionic bonds
 Encapsulated  Van der Waals
 Divided into 3 regions  Hydrophobic interactions
o Cortex – primary follicles (B cells, - Univalent interaction
macrophages, dendritic cells) - Bivalent interaction
o Paracortex – T cells, dendritic cells  Affinity
o Medulla - plasma cells secreting - As an immune response progresses, cells that
antibody have been activated will produce and express
- Lymph nodes more receptors
 Primary follicle – unactivated lymphoid - The receptors have higher affinity as the immune
follicle response progresses
 Secondary follicle  Receptor locations
o Activated by antigen - Immune antigen receptors can be transmembrane,
o Germinal center – area of activated B cytosolic or secreted
cells  Antibody can be secreted or transmembrane
- Spleen (B-cell receptor)
 Filters blood, traps blood-borne pathogens  T-cell receptor is transmembrane
 Important in systemic infections  Some innate immune system receptors are
 Blood enters through splenic artery cytosolic (we have receptors in our cells that
 Encapsulated might recognize viral particles)
 Structure
o Projections from capsule from trabeculae Adaptive Immune System Receptors
o Compartments  B-cell receptors
 Red pulp – macrophages, RBCs - B cells produce antibodies (Abs) that are secreted
 White pulp - Some antibodies are membrane bound and are
 Surrounds branches of splenic associated with other proteins
artery - Collectively known as the B-cell receptor (TCR)
 Forms PALS (periarteriolar which is membrane bound
lymphoid sheath)  Antibodies and T-cell receptors
 Primary follicles rich in B cells - Recognizes epitopes
- MALT (Mucosa-associates Lymphoid Tissue) - Immunologically active regions of immunogen
 Can be specified further that bind to antigen-specific antibodies or T-cell
o Gut-associated lymphoid tissues (GALT) receptors
o Bronchus-associated lymphoid tissue  Epitopes
(BALT) - Antigenic determinants recognized by B cells and
 Organized areas along digestive, respiratory, T cells
and urogenital tracts - B cell epitopes tend to be on the outside of the
 Very well organized areas in intestine are antigen
referred to as Peyer’s patches  For example, the hydrophilic amin acids on a
 Includes tonsils and appendix protein’s surface
- T cell epitopes from proteins derived from
Immune Recognition and Response enzymatic digestion of peptide and then
association with MHC
Introduction - Most antigens offer multiple epitopes
 Cells of the immune system must constantly recognize - However, a single B cell will only produce
molecular signals from the external environment antibody specific to single epitope
- The immune cells use several different receptors
to do so:
 Adaptive immune cells (B and T cells) – use
specific B-cell receptors (BCRs) and T-cell
receptors (TCRs)
dane.
Antibodies (Abs)
 Epitope-binding proteins
- Membrane bound on B cells (as part of the B-cell
receptor) OR  Overall structure of Immunoglobulin
- Secreted in blood (Humoral immunity) - Primary – sequence of amino acids
- Secondary – folding into series of β-pleated
 Share structural features, bind to antigen, and
sheets
participate in number of effector functions
- Tertiary – compact globular domains
 Known collectively as Immunoglobulins (Igs)
- Quarternary – adjacent light and heavy chains
 Antibodies don’t kill anything, their job is to serve as
interact
target markers for other immune system components
 Hypervariable Regions
 Basic structure
- Complimentarily-determining regions (CDRs)
- Known since late 19th century that antibodies are
- Complimentary to epitopes that they will bind to
in serum
 Hinge region
 Serum is fluid phase that remains after
- ɣ (gamma), δ (delta), and α (alpha) heavy chains
plasma is allowed to clot
have extended peptide sequence
 Antibodies are also found in other secretions
- Rich in proline and cysteine
- Heterodimers
- Gives flexibility
 2 light chains – 22,000 daltons each
 B Cell Receptor (BCR)
 2 heavy chains – 55,000 daltons each
- Immunoglobulins can be secreted or membrane-
 First 110 aa of amino terminal end of heavy
bound
and light chain vary depending on antibody
- Membrane-bound differs from secreted in the
specificity
carbonyl-terminal end:
- Light chains
 Extracellular “spacer” of 26 aa
 When aa sequences of light chains from
 Hydrophobic transmembrane sequence
several individuals were sequenced, pattern
 Cytoplasmic tail
emerged:
- Membrane-bound antibody + accessory
o Amino terminal end (110 aa) varies
membrane-bound molecules = BCR
o Other part remained constant
- Heavy chain portion of membrane-bound
 Were found to be either kappa (κ) or
antibody does not extend far enough through the
lambda (λ)
cell membrane for signaling
 In mice and humans, different
- Also associated with CD21, CD19, and CD81
lambda subtypes have been
found  Antibody-mediates effector functions
- Heavy chains - Remember, they “mark” an invader
 Amino-terminal end also shows variability - In addition to binding antigen, Abs can:
 Each antibody has 2 identical heavy chains, 2  Promote phagocytosis (opsonization)
identical light chains  Activate complement
 5 different heavy chain constant regions  Antibody dependent cell mediated toxicity
(isotypes) (ADCC)
o IgM – µ  NK cells have receptor for Fc portion of
o IgG – ɣ antibody
o IgA – α  Some can cross epithelial layers to be
o IgD – δ excreted through mucous or across placenta
o IgE – ε  Antibody class switching
 Some subisotypes have been discovered in - Changing the constant region of an antibody is
some species class switching
dane.
- Makes an IgG from an IgM  IgA
- Predominant class in secretions
Antibody Isotypes - J chain and secretory component helps with
 IgM transport across intestinal wall
- Monomeric IgM expressed on B cells - J chain makes IgA more resistant to acids and
- Secreted is pentameric enzymes found in digestive tract
- 1st class produced in primary response - IgA and macrophages restrict commensal bacteria
- Activates complement that occasionally enter the tissues from the
- Very good at agglutination intestines
- Better for IgA than IgG to interact with
macrophage—because the Fc portio of IgG has
high affinity for receptors of immune cells and
would trigger inflammatory responses
- Exists as a dimer
- Can cross-link large antigens

 IgD
- Membrane bound on B cells

 Iso-, Allo-, and Idiotype

 IgG
- Most abundant
- 4 human subclasses
- Crosses placenta
- Involved in complement activation
- Opsonin
- IgG subclasses

T-Cell Receptor
 Expressed on surface of T-cells
 Recognize processed antigen complexed with MHC
molecules
 IgE  T cell VS B Cell Receptor
- Involved in allergic reactions - T cell receptor is only membrane bound – doesn’t
- Involvement in parasitic infections appear in soluble form like antibodies so more
difficult to assess its structure
- Antigen binding of T cell receptor is weaker than
that of antibodies
- Antigen recognized by T cells is not antigen alone
but antigen associated with MHC molecules
 TCR-MHC interaction
- (a) T cell receptor is specific for peptide A
- (b) Right MHC haplotype but wrong antigen
(peptide B)
- (c) Right antigen (peptide A) but wrong haplotype

dane.
domain wall
(NOD)- peptidoglycans
like
receptor
(NLR)
Absent- Cytosol and Viral and Production of
in- nucleus bacterial DNAs interferons
melanom and cytokines
a (AIM)-
like
 TCR heterodimers are similar to immunoglobulins receptor
 Therefore they are classified in immunoglobulin (ALR)
superfamilt  Cytokines
 Resembles Fab portion of antibody - Low-molecular weight regulatory proteins or
 TCR Coreceptors glycoproteins
- Accessory molecules help in signal transduction - Secreted by and through which WBC and various
after interaction of T cell with antigen (e.g., CD4, other cells communicate
CD8, CD28) - Assist in development of immune effector cells
- Assist with signal transduction - Some possess direct effector functions of their
- Affinity of TCR for peptide MHC complexes is own
enhanced by coreceptors - Some referred to as interleukins (IL)
- IL-1 through IL-29 have been described
Innate Immune System Receptors - Chemokines
 Remember that Abs and TCRs recognize specific  Subpopulation of cytokines
epitopes  Mobilize immune cells from one area to
 Innate immune system receptor are Pattern another (help move WBCs into tissues)
Recognition Receptors (PRRs) that recognize  Class of chemoattractants
Pathogen Associated Molecular Patterns (PAMPS) - Cytokine action
 Pattern Recognition Receptors (PRRs)  Mainly functions by autocrine and paracrine
Full Cellular Ligands Cellular action
name locations functions
Toll-like Plasma Microbial Production of
receptor membrane, carbohydrates, antimicrobials,
(TLR) endosomes, lipoproteins, antivirals, and
lysosomes fungal cytokines;
mannans, inflammation
bacterial
flagelline, viral
RNA, self-
components of
damaged
tissues, etc  Cytokines bind to specific receptors
C-type Plasma Carbohydrate Phagocytosis,  Trigger signal transduction pathways that
lectin membrane components of production of alter gene expression in target cells
receptor fungi, antimicrobials
(CLR) mycobacteria, and cytokines;
viruses, inflammation
parasites, and
some allergens
Retinoic Cytosol Viral RNA Production of
acid- interferons
inducible and cytokines
gene-I
(RIG-I)-
like
receptor
Nucleoti Cytosol Fragments of Production of
de intracellular or antimicrobials  Exhibit pleiotropy, redundancy, synergy,
oligomer extracellular and cytokines; antagonism, cascade induction
ization bacteria cell inflammation

dane.
 Non-specificity of cytokines contradicts with o Bovine serum albumin (BSA) is not
established specificity of immune system immunogenic when injected into cow but
it is when injected into chicken
 How does this work?  Some macromolecules are highly conserved
o Careful expression of the receptors for throughout evolution and display little
those cytokines on specific cells immunogenicity
o Receptors are often only expressed after o Cytochrome C and collagen
exposure to antigen - Molecular size
- Cytokine Families  Active (good) immunogens - >100,000
 Cytokines belong to 6 families daltons
o Interleukin 1 (IL-1)  Poor immunogens - <5,000-10,000 daltons
o Hematopoietin family (Class i) - Chemical composition and complexity
o Interferon family (Class II)  Polymers composed of multiple copies of
o Chemokine family same amino acid or sugar tend to be poor
o Tumor necrosis family immunogens
o Interleukin 17 (IL-17)  Lipids are haptens and need to be conjugated
 Based on structural studies with carrier to produce antibodies –
 All have molecular mass less than 30kDa important for assays for detection of some
 All have similarities and few rarely act alone steroids and vitamins
- Cytokine Receptors - Ability to be processes and presented on MHC
 Cytokine receptors fall into families  Susceptibility to antigen processing
o Immunoglobulin superfamily receptor  Genotype of recipient animal
o Class I cytokine receptor family  Material presentation
(hematopoietin)  Susceptibility to Antigen Processing
o Class II cytokine receptor family - Large, insoluble macrophages are more likely to
(interferon) be phagocytized for processing
o TNF receptor family  Genotype of recipient animal
o Chemokine receptor family - Genes of MHC
 IL-2R is the most thoroughly studied cytokine - Genes encoding for specific antibodies
receptor  Material presentation
o Expressed by T cells for proliferation - Immunogen dosage
(also called CD25, surface marker in  Too low or high of dosage can induce
mature T cells) tolerance
o Complete receptor has 3 subunits –  Single dose is often not enough, booster is
subunits together have higher affinity for needed
IL-2 - Route of administration
- Cytokine-Cytokine Receptor Interaction  Intravenous (IV) – travel to spleen
 Initiates a signal cascade in the cell resulting  Intradermal (ID)
in a nuclear response (activation of  Subcutaneous (SC) – travel to lymph nodes
transcription factors resulting in production  Intramuscular (IM)
of proteins, resulting in mitosis/proliferation,  Intraperitoneal (IP)
etc.)  Adjuvants
- Enhance immunogenicity
Immunogen VS Antigen - Not exactly sure how they work but are
 Immunogenicity recognized by Toll-like receptors
- Ability to induce humoral (B cell antibody) and/or - Water-in-oil adjuvants
cell-mediates (T cell) immune response  Freund’s incomplete adjuvant
- Immunogen is the substance that induces - Antigen in aqueous solution, mineral oil, and
response emulsifying agent
 Antigenicity - Antigen is then released very slowly from
- Ability to combine specifically with Abs or T-cell injection site
receptor/MHC - Based on Freund’s complete adjuvant which
- Not all antigens are immunogenic (e.g., haptens) contains heat-killed Mycobacteria
 Properties of Immunogen contribute to  Haptens
immunogenicity - Too small, lack immunogenicity
- Foreignness - If hapten is coupled to carrier protein, immune
 Lymphocytes that do not bind to self-antigens response can be induced
are allowed to further develop  Hapten-Carrier conjugate
 Therefore, they will later only recognize non- - Produces 3 types of antigenic determinants
self antigens  Antibodies to hapten
 For example:
dane.
 Antibodies to carrier
 Antibodies to hapten-carrier conjugate

dane.

You might also like